ORTHOPEDIC MCQS WITH ANSWER UPPER LIMB 05
ORTHOPEDIC MCQS WITH ANSWER UPPER LIMB 05
1. A 36-year-old woman has pain and swelling of the anterior arm after undergoing arthroscopic shoulder surgery 8 months ago. At the time of the procedure, extensive debridement and synovectomy of the anterior aspect of the joint was performed to remove scar tissue that had formed after an open rotator cuff repair. Examination reveals a golf ball-sized swelling just lateral to the coracoid. The area is not warm and shows no other signs of infection. An MRI scan is shown in Figure 1. Management should now consist of
1- aspiration of the ganglion cyst.
2- repair of the supraspinatus tendon.
3- repair of the subscapularis tendon.
4- repair of the rotator cuff interval.
5- repair of the anterior labrum.
PREFERRED RESPONSE: 4
DISCUSSION: Deficiency of the rotator cuff interval may be acquired or congenital. In this patient, extensive debridement of the rotator cuff interval capsule at the time of arthroscopy most likely is the cause of the defect seen on the MRI scan. Surgical closure of the defect is the treatment of choice. During the repair, the shoulder should be placed in 30 degrees of external rotation to avoid overtightening. Care should be taken to include the leading edge of both the supraspinatus and subscapularis tendons in the repair because the rotator cuff interval capsular tissue is likely to be of poor quality.
REFERENCES: Cole BJ, Rodeo SA, O’Brien SJ, et al: The anatomy and histology of the rotator interval capsule of the shoulder. Clin Orthop 2001;390:129-137.
Jost B, Koch PP, Gerber C: Anatomy and functional aspects of the rotator interval. J Shoulder Elbow Surg 2000;9:336-341.
2. A 45-year-old recreational tennis player underwent arthroscopic decompression and mini-open repair of a small supraspinatus tendon tear 3 weeks ago after nonsurgical management failed to provide relief. He now has pain, swelling about the wound, erythema, and purulent drainage. The patient is returned to the operating room for irrigation, debridement, and cultures. What is the most common organism causing
this infection?
1- Staphylococcus epidermidis
2- Methicillin-resistant Staphylococcus aureus
3- Pseudomonas aeruginosa
4- Propionibacterium acnes
5- Clostridium tetani
PREFERRED RESPONSE: 4
DISCUSSION: In a large series of mini-open rotator cuff repairs, an infection rate of at least 2% was found, with the majority of the infections caused by Propionibacterium acnes. To prevent this complication, the shoulder should be re-prepped before the mini-open incision is made to prevent bacterial contamination from the arthroscopic procedure.
REFERENCES: Herrera MF, Bauer G, Reynolds F, et al: Infection after mini-open rotator cuff repair. J Shoulder Elbow Surg 2002;11:605-608.
Settecerri JJ, Pitner MA, Rock MG, et al: Infection after rotator cuff repair. J Shoulder Elbow Surg 1999;8:1-5.
3. A paraplegic 32-year-old man was pulling himself up in bed by grasping the headboard rails when he felt a pop and immediate pain. A radiograph and CT scan are shown in Figures 2a and 2b. Based on these findings, management should consist of
1- rest and a sling for 3 to 6 weeks.
2- physical therapy with modalities for pain relief.
3- pain medication and activity as tolerated.
4- open reduction and internal fixation.
5- coracoid excision.
PREFERRED RESPONSE: 4
DISCUSSION: The coracoid process is an essential component of the superior shoulder suspensory complex and must be maintained. Open reduction and internal fixation is recommended if the fragment is large and displaced more than 1 cm.
REFERENCES: Froimson AI: Fracture of the coracoid process of the scapula. J Bone Joint Surg Am 1978;60:710-711.
Gil JF, Haydar A: Isolated injury of the coracoid process: Case report. J Trauma
1991;31:1696-1697.
4. A 23-year-old baseball pitcher reports pain in the posterior aspect of his dominant shoulder during the late cocking phase of throwing. With the dominant shoulder positioned in 90 degrees of abduction from the body and with the scapula stabilized, examination reveals 135 degrees of external rotation and 20 degrees of internal rotation. Examination of the opposite shoulder reveals 100 degrees of external rotation and 75 degrees of internal rotation. Both shoulders are stable on examination. Radiographs and MRI scans are unremarkable. What is the primary cause of his pain?
1- Posterior capsular tightness
2- Tightness of the rotator cuff interval
3- Anterior inferior glenohumeral ligament laxity
4- Excessive retroversion of the proximal humerus
5- Subacromial impingement
PREFERRED RESPONSE: 1
DISCUSSION: Internal impingement of the shoulder is a leading cause of shoulder pain in the throwing athlete. The primary lesion in pathologic internal impingement is excessive tightening of the posterior band of the inferior glenohumeral ligament complex. To obtain an accurate assessment of true glenohumeral rotation, the scapula is stabilized during examination. A loss of 20 degrees or more of internal rotation, as measured with the shoulder positioned in 90 degrees of abduction, indicates excessive tightness of the posterior band of the inferior glenohumeral ligament complex.
REFERENCES: Burkhart SS, Morgan CD, Kibler WB: The disabled throwing shoulder: Spectrum of pathology. Part I: Pathoanatomy and biomechanics. Arthroscopy 2003;19:404-420.
Meister K: Injuries to the shoulder in the throwing athlete. Part one: Biomechanics, pathophysiology, classification of injury. Am J Sports Med 2000;28:265-275.
5. What is the most common indication for revision following unconstrained
elbow arthroplasty?
1- Polyethylene wear
2- Loosening of the humeral component
3- Loosening of the ulnar component
4- Instability
5- Component failure
PREFERRED RESPONSE: 4
DISCUSSION: Instability following unconstrained elbow arthroplasty occurs in 10% of patients. Subluxation is twice as common as frank dislocation; however, only 20% of these patients undergo revision. Instability following unconstrained elbow arthroplasty can be caused by component malposition or ligament insufficiency.
REFERENCES: King GJ, Itoi E, Niebur GL, et al: Motion and laxity of the capitellocondylar total elbow prosthesis. J Bone Joint Surg Am 1994;76:1000-1008.
Ring D, Koris M, Jupiter JB: Instability after total elbow arthroplasty. Orthop Clin North Am 2001;32:671-677.
6. What part of the glenoid labrum has the least vascularity?
1- Midanterior
2- Anterior/inferior
3- Anterior/superior
4- Posterior/superior
5- Posterior/inferior
PREFERRED RESPONSE: 3
DISCUSSION: The glenoid labrum receives its blood supply from the suprascapular, posterior humeral circumflex, and circumflex scapular arteries. The labral vessels arise from the capsular and periosteal vessels that penetrate the periphery of the labrum. The bone does not appear to be a source of vascularity. The posterior/superior and inferior labrum have a fairly robust vascular supply, whereas the anterior/superior labrum has relatively poor vascularity, which may influence the success of superior lateral repairs.
REFERENCE: Cooper DE, Arnoczky SP, O’Brien SJ, et al: Anatomy, histology and vascularity of the glenoid labrum: An anatomical study. J Bone Joint Surg Am 1992;74:46-52.
7. One of the serious potential complications of repair of distal biceps tendon ruptures is limited pronation and supination as a result of synostosis. What surgical approach and technique presents the highest risk for development of this complication?
1- Single incision, anterior approach with fixation through drill holes in the radius
2- Single incision, anterior approach with suture anchor fixation to the radius
3- Single incision, anterior approach through a drill hole in the radius with sutures tied over bolster or button on the posterior forearm
4- Dual incision, limited anterior and posterior approach along the ulna with attachment through drill holes
5- Dual incision, limited anterior and posterior muscle-splitting approach (supinator and extensor muscles) with attachment through drill holes
PREFERRED RESPONSE: 4
DISCUSSION: The risk of synostosis is imminent with any technique for repairing a distal biceps tendon rupture. However, the risk is quite low for all approaches that avoid exposure of the ulna, including the muscle-splitting two-incision technique.
REFERENCE: Norris TR: Orthopaedic Knowledge Update: Shoulder and Elbow. Rosemont, IL, American Academy of Orthopaedic Surgeons, 1997, p 342.
8. A 25-year-old carpenter falls on his outstretched arm. What physical finding best correlates with the lesion seen on the MRI scan shown in Figure 3?
1- Weakness in external rotation
2- Weakness in abduction
3- Positive lift-off test
4- Loss of biceps contour
5- Deltoid atrophy
PREFERRED RESPONSE: 3
DISCUSSION: The MRI scan shows disruption of the subscapularis muscle. Subscapularis rupture is associated with weakness in internal rotation as shown with a positive lift-off test as described by Gerber and Krushell. The belly press test also has been shown to be a useful clinical test for this problem. Weakness in external rotation and abduction is more consistent with supraspinatus and infraspinatus tears. Deltoid atrophy is associated with an axillary nerve injury. Loss of biceps contour is associated with rupture of the long head of the biceps.
REFERENCES: Gerber C, Krushell RJ: Isolated rupture of the tendon of the subscapularis muscle: Clinical features in 16 cases. J Bone Joint Surg Br 1991;73:389-394.
Greis PE, Kuhn JE, Schultheis J, et al: Validation of the lift-off test and analysis of subscapularis activity during maximal internal rotation. Am J Sports Med 1996;24:589-593.
9. A 72-year-old woman who sustained a cerebrovascular accident 9 months ago now has a fixed elbow flexion contracture of 80 degrees. Management should consist of
1- passive physical therapy.
2- musculocutaneous neurectomy and serial casting.
3- musculocutaneous nerve block.
4- lengthening of the biceps tendon.
5- distraction arthroplasty.
PREFERRED RESPONSE: 2
DISCUSSION: A flexion contracture of the elbow is commonly seen in hemiplegic patients following cerebrovascular accidents. Spasticity and myostatic contracture of the joint are both causative factors. In patients with a flexion deformity of less than 90 degrees, musculocutaneous neurectomy is recommended, followed by serial casting to treat any residual deformity. At 9 months after injury, physical therapy will not significantly improve motion. Nerve blocks may be used in the early stages of recovery to facilitate therapy and serial casting.
REFERENCE: Waters RL, Keenan ME: Surgical treatment of the upper extremity after stroke, in Chapman MW (ed): Operative Orthopedics. Philadelphia, PA, JB Lippincott, 1988, vol 2,
pp 1449-1450.
10. A 44-year-old recreational weight lifter reports chronic deep pain in his left shoulder that is aggravated by any pressing exercises. He also notes a painful catch in the shoulder occurring with rotational movements. Physical therapy and nonsteroidal anti-inflammatory drugs for 3 months have failed to provide relief. Examination reveals pain with O’Brien’s test but no signs of instability. MRI scans are shown in Figures 4a
and 4b. Treatment should now consist of
1- arthroscopic repair of a superior labral tear with cyst decompression.
2- open excision of the ganglion cyst.
3- proximal biceps tenodesis.
4- rotator cuff repair.
5- anterior stabilization.
PREFERRED RESPONSE: 1
DISCUSSION: The MRI scans show a large paralabral ganglion cyst in the spinoglenoid notch that communicates with an extensive tear of the glenoid labrum. Snyder and associates have classified superior labral tears into several subtypes that reflect the location and extent of the injury. Arthroscopic repair of the labral tear and aspiration of the ganglion cyst is the treatment of choice. Open excision of the cyst does not address the underlying problem of the labral tear.
REFERENCE: Snyder SJ, Karzel RP, Delpizzo W: SLAP lesions of the shoulder. Arthroscopy 1990;6:274-279.
Piatt BE, Hawkins RJ, Fritz RC, et al: Clinical evaluation and treatment of spinoglenoid notch cysts. J Shoulder Elbow Surg 2002;11:600-604.
McFarland EG, Kim TK, Savino RM: Clinical assessment of three common tests for superior labral anterior-posterior lesions. Am J Sports Med 2002;30:810-815.
O’Brien SJ, Pagnani MJ, Fealy S, et al: The active compression test: A new and effective test for diagnosing labral tears and acromioclavicular joint abnormality. Am J Sports Med
1998;26:610-613.
11. A 35-year-old carpenter sustained an injury to his dominant shoulder in a fall. He reports that he felt a sharp tearing sensation as he held on to a scaffold to keep from falling. Examination reveals swelling and ecchymosis down the upper arm, weakness to internal rotation, and deformity of the anterior axilla. He has good strength in external rotation and no apprehension with instability testing. Radiographs are normal. Management should consist of
1- physical therapy for range of motion and strengthening following a decrease in pain and swelling.
2- shoulder arthroscopy for diagnosis and treatment of a possible labral tear.
3- open exploration and repair of a pectoralis major tendon avulsion.
4- MRI of the rotator cuff.
5- immobilization in a sling and swathe for 3 to 6 weeks, followed by mobilization and strengthening.
PREFERRED RESPONSE: 3
DISCUSSION: The findings are classic for a pectoralis major tendon avulsion. Deformity of the anterior axillary fold is a classic finding, and ecchymosis down the arm suggests that the injury is at the humeral attachment rather than at the musculotendinous junction. Good external rotation strength indicates that function in the supraspinatus and infraspinatus has been preserved. The treatment of choice for a tendon avulsion in a young individual is early surgical repair. Conversely, if the injury is within the muscle or at the musculotendinous junction, initial nonsurgical management is recommended. If the location of the injury cannot be determined by physical examination, then MRI of the pectoralis major can be helpful.
REFERENCES: Hanna CM, Glenny AB, Stanley SN, et al: Pectoralis major tears: Comparison of surgical and conservative treatment. Br J Sports Med 2001;35:202-206.
Connell DA, Potter HG, Sherman MF, et al: Injuries of the pectoralis major muscle: Evaluation with MR imaging. Radiology 1999;210:785-791.
12. A 48-year-old woman with rheumatoid arthritis reports increasing elbow pain for the past 6 months. History reveals that she underwent total elbow arthroplasty 7 years ago. A peripheral WBC count, erythrocyte sedimentation rate, and C-reactive protein studies are normal. An AP radiograph is shown in Figure 5. What is the next most appropriate step in management?
1- Continued observation
2- Synovectomy and bushing replacement
3- Reimplantation of both the humeral and ulnar components
4- Conversion to a resection arthroplasty
5- Conversion to elbow arthrodesis
PREFERRED RESPONSE: 2
DISCUSSION: Pain relief is excellent after total elbow arthroplasty and is comparable to the results found with hip and knee arthroplasty. The failure of total elbow arthroplasty in the treatment of rheumatoid arthritis can be the result of infection, aseptic loosening, instability, and bearing surface wear. The radiographic findings shown here are consistent with bushing wear in a linked device. The bushings can be changed before continued wear results in osteolysis and implant loosening. If the implants become loose, then reimplantation is necessary. Resection arthroplasty is not indicated if the components are well fixed. Elbow arthrodesis is not indicated in patients with rheumatoid arthritis.
REFERENCES: Gill DR, Morrey BF: The Coonrad-Morrey total elbow arthroplasty in patients who have rheumatoid arthritis: A ten to fifteen-year follow-up study. J Bone Joint Surg Am 1998;80:1327-1335.
Norris T (ed): Orthopaedic Knowledge Update: Shoulder and Elbow 2. Rosemont, IL, American Academy of Orthopaedic Surgeons, 2002, pp 333-341.
13. Figure 6a shows the radiograph of a 50-year-old man who sustained an anterior dislocation of the shoulder. He undergoes closed reduction, and the postreduction radiograph is shown in Figure 6b. Management should now consist of
1- continued use of a sling for 3 to 4 weeks, followed by repeat radiographs.
2- open reduction and internal fixation of the greater tuberosity fracture.
3- repeat reduction and placement of an abduction orthosis.
4- hemiarthroplasty.
5- percutaneous pinning.
PREFERRED RESPONSE: 2
DISCUSSION: Displaced greater tuberosity fractures often will block abduction and/or external rotation by impinging on the underside of the acromion or posterior glenoid. The indications for open reduction and internal fixation are 1 cm of displacement or 45 degrees of rotation of the tuberosity fracture. Surgical treatment has recently been recommended for 0.5 cm of tuberosity displacement.
REFERENCES: Neer CS II: Displaced proximal humeral fractures: II. Treatment of three-part and four-part displacement. J Bone Joint Surg Am 1970;52:1090-1103.
Flatow EL, Cuomo F, Maday MG, et al: Open reduction and internal fixation of two-part displaced fractures of the greater tuberosity of the proximal part of the humerus. J Bone Joint Surg Am 1991;73:1213-1218.
14. A 42-year-old man sustained a fracture of the distal radius with subsequent stiffness in the ipsilateral shoulder. Despite a 6-month program of range-of-motion exercises, external rotation at the side is limited to 10 degrees. Attempts at closed manipulation are unsuccessful. Treatment should now consist of
1- open release of the posterior capsule.
2- arthroscopic release of the rotator cuff interval.
3- arthroscopic release of the anteroinferior capsule.
4- open subscapularis lengthening.
5- open extra-articular release.
PREFERRED RESPONSE: 2
DISCUSSION: When external rotation at the side is limited, the most likely diagnosis is contracture of the rotator cuff interval, including the superior glenohumeral and coracohumeral ligaments. Therefore, the treatment of choice is arthroscopic release of the rotator cuff interval.
REFERENCE: Harryman DT II, Matsen FA III, Sidles JA: Arthroscopic management of refractory shoulder stiffness. Arthroscopy 1997;13:133-147.
15. A 72-year-old woman who fell on her right shoulder while using a treadmill is now unable to elevate her right arm. An MRI scan is shown in Figure 7. What is the most likely diagnosis?
1- Axillary nerve injury
2- Anterior dislocation
3- Extension of a chronic large rotator cuff tear
4- Suprascapular nerve entrapment from a ganglion cyst
5- Greater tuberosity avulsion
PREFERRED RESPONSE: 3
DISCUSSION: The MRI scan reveals a large chronic rotator cuff tear with retraction and fatty infiltration atrophy of the supraspinatus and infraspinatus tendons. This tear is responsible for the patient’s severe weakness and inability to elevate the arm.
REFERENCE: Gerber C, Myer DC, Schneeberger AG, et al: Effect of tendon release and delayed repair on the structure of the muscles of the rotator cuff: An experimental study in sheep. J Bone Joint Surg Am 2004;86:1973-1982.
16. A 45-year-old man who underwent an open capsulolabral stabilization procedure
15 years ago now reports pain and has no external rotation on the affected side. Nonsurgical management has failed to provide relief. Examination reveals external rotation to -5 degrees compared with 50 degrees of external rotation on the contralateral side. Radiographs show a small inferior osteophyte and minimal posterior glenoid wear. Which of the following procedures will offer the best chance of restoring motion, decreasing pain, and preserving the native joint?
1- Arthroscopic removal of the osteophytes
2- Arthroscopic debridement and acromioplasty
3- Arthroscopic release of the posterior capsule
4- Release of the rotator interval and anterior capsule
5- Closed manipulation under anesthesia
PREFERRED RESPONSE: 4
DISCUSSION: Loss of external rotation following stabilization procedures can result in progressive degenerative joint disease. A tight anterior capsule results in posterior humeral translation and progressive posterior glenoid wear. Patients with early degenerative joint disease and pain can be treated with anterior release to restore more normal glenohumeral biomechanics. This procedure not only improves function but also decreases pain in most patients. Closed manipulation at 15 years after surgery is unlikely to be successful and carries the risk of complications. Acromioplasty, posterior release, and removal of osteophytes do not address the pathology. Arthroscopic releases are favored for intra-articular procedures that have addressed the pathology of instability. Open releases are recommended for nonanatomic extra-articular repairs that include subscapularis tightening procedures.
REFERENCES: MacDonald PB, Hawkins RJ, Fowler PJ, et al: Release of the subscapularis for internal rotation contracture and pain after anterior repair for recurrent anterior dislocation of the shoulder. J Bone Joint Surg Am 1992;74:734-737.
Warner JJ, Allen AA, Marks PH, et al: Arthroscopic release of postoperative capsular contracture of the shoulder. J Bone Joint Surg Am 1997;79:1151-1158.
17. A right-handed 44-year-old construction worker reports pain and limited range of motion in his right elbow that has limited his ability to work for the past year. Examination reveals range of motion from 60 to 90 degrees, and he has pain at the extremes of flexion and extension. Pronation and supination are minimally restricted. Anti-inflammatory drugs have failed to provide relief. A radiograph is shown in Figure 8. Management should now consist of
1- observation.
2- physical therapy.
3- ulnohumeral arthroplasty.
4- ulnohumeral arthroplasty and ulnar nerve decompression.
5- semiconstrained elbow arthroplasty.
PREFERRED RESPONSE: 4
DISCUSSION: The radiograph reveals primary osteoarthritis of the elbow; therefore, ulnohumeral arthroplasty is the preferred procedure. Patients with severely limited preoperative elbow extension of more than 60 degrees and flexion of less than 100 degrees are at risk for ulnar nerve dysfunction postoperatively and should undergo a concomitant ulnar nerve decompression. Nonsurgical methods are unlikely to improve his chronic condition. Elbow arthroplasty is contraindicated for patients in this age group and with this diagnosis.
REFERENCES: Antuna SA, Morrey BF, Adams RA, et al: Ulnohumeral arthroplasty for primary degenerative arthritis of the elbow: Long-term outcome and complications. J Bone Joint Surg Am 2002;84:2168-2173.
Phillips NJ, Ali A, Stanley D: Treatment of primary degenerative arthritis of the elbow by ulnohumeral arthroplasty: A long-term follow-up. J Bone Joint Surg Br 2003;85:347-350.
18. Spontaneous recovery of upper extremtiy motor function after a cerebrovascular accident occurs in which of the following predictable patterns?
1- Shoulder flexion, elbow flexion, wrist flexion, finger flexion
2- Shoulder flexion, forearm supination, wrist flexion, finger flexion
3- Shoulder extension, elbow extension, wrist extension, finger extension
4- Finger flexion, wrist extension, elbow flexion, shoulder flexion
5- Finger flexion, wrist flexion, elbow flexion, shoulder flexion
PREFERRED RESPONSE: 1
DISCUSSION: Recovery of upper extremity motor function after a cerebrovascular accident follows a predictable pattern. The greatest amount of recovery is seen within the first 6 weeks. Return of function proceeds from proximal to distal. Shoulder flexion occurs first, followed by return of flexion to the elbow, wrist, and fingers. Return of forearm supination follows the return of finger flexion.
REFERENCE: Waters RL, Keenan ME: Surgical treatment of the upper extremity after stroke, in Chapman MW (ed): Operative Orthopedics. Philadelphia, PA, JB Lippincott, 1988, vol 2,
pp 1449-1450.
19. A 65-year-old woman sustained an axial load on the arm followed by an abduction injury after falling on ice. Treatment in the emergency department consisted of reduction of an anterior dislocation. She now has a positive drop arm sign and a positive lift-off test. An MRI scan is shown in Figure 9. Based on these findings, management should consist of
1- tenolysis of the biceps.
2- repair of the subscapularis using suture anchors.
3- repair of the subscapularis tendon and biceps tenodesis.
4- repair of the subscapularis tendon and removal of the loose body.
5- observation.
PREFERRED RESPONSE: 3
DISCUSSION: Dislocation of the long head of the biceps tendon is the result of a defect in the region of the rotator cuff interval, coracohumeral ligament-superior glenohumeral ligament pulley, or an associated tear of the medial insertion of the subscapularis tendon. In the case of an intra-articular dislocation of the long head of the biceps tendon associated with a tear of the subscapularis tendon, stabilization of the biceps tendon is difficult in this situation; therefore, biceps release or tenodesis and repair of the subscapularis tendon is the treatment of choice.
REFERENCES: Eakin CL, Faber KJ, Hawkins RJ, et al: Biceps tendon disorders in athletes.
J Am Acad Orthop Surg 1999;7:300-310.
Sethi N, Wright R, Yamaguchi K: Disorders of the long head of the biceps tendon. J Shoulder Elbow Surg 1999;8:644-654.
Walch G, Boileau P: Subluxations and dislocations of the tendon of the long head of the biceps. J Shoulder Elbow Surg 1998;7:100-108.
20. What type of nerve palsy is most common following elbow arthroscopy?
1- Transient posterior interosseous
2- Transient ulnar
3- Transient radial
4- Transient median
5- Transient medial antebrachial cutaneous
PREFERRED RESPONSE: 2
DISCUSSION: Transient ulnar nerve palsy is the most common palsy following elbow arthroscopy. The ulnar nerve is most frequently affected, followed by the radial nerve. Injury to the other nerves has been reported but less frequently.
REFERENCES: Kelly EW, Morrey BF, O’Driscoll SW: Complications of elbow arthroscopy.
J Bone Joint Surg Am 2001;83:25-34.
Morrey BF: Elbow complication, in Morrey BF (ed): The Elbow and Its Disorders, ed 3. Philadelphia, PA, WB Saunders, 2001, pp 519-522.
21. A 72-year-old man injured his right shoulder after tripping over a chair leg. Radiographs obtained in the emergency department reveal an isolated anterior dislocation. After successful closed reduction, the patient has recurrent anterior instability and is unable to elevate the arm. What is the most likely cause of the recurrent instability?
1- Infection of the anterior glenoid labral detachment
2- Anterior glenoid fracture
3- Axilllary nerve palsy
4- Occult surgical neck fracture
5- Rotator cuff tear
PREFERRED RESPONSE: 5
DISCUSSION: A rotator cuff tear is the most common cause of recurrent instability following a first-time dislocation in patients older than age 40 years. Dislocations occur through a posterior mechanism rather than by an isolated labral avulsion or a Bankart lesion as seen in younger patients.
REFERENCES: Nevaiser RJ, Nevaiser TJ: Recurrent instability of the shoulder after age 40.
J Shoulder Elbow Surg 1995;4:416-418.
Pevny T, Hunter RE, Freeman JR: Primary traumatic anterior shoulder dislocation in patients 40 years of age and older. Arthroscopy 1998;14:289-294.
22. A 54-year-old woman sustained an elbow injury 3 months ago that was treated with open reduction and internal fixation. She now reports pain and limited elbow motion. Radiographs are shown in Figures 10a and 10b. Treatment should now consist of
1- occupational therapy.
2- open reduction of the radial head and annular ligament reconstruction.
3- excision of the radial head.
4- ulnar osteotomy and closed reduction of the radial head.
5- ulnar osteotomy and open reduction of the radial head.
PREFERRED RESPONSE: 5
DISCUSSION: Radiographs reveal malunion of a Monteggia fracture-dislocation. Dislocation of the posterior radial head is caused by the malunited ulnar fracture. The deformity includes shortening with an apex posterior angulation. In the acute setting, open reduction of the radial head rarely is necessary; however, in chronic dislocations, open reduction is required. Without ulnar osteotomy, recurrent radial head dislocation is likely.
REFERENCE: Horii E, Nakamura R, Koh S, et al: Surgical treatment for chronic radial head dislocation. J Bone Joint Surg Am 2002;84:1183-1188.
23. Ulnohumeral distraction interposition arthroplasty is considered the most appropriate treatment for which of the following patients?
1- A 41-year-old man with painful posttraumatic arthritis of the elbow
2- A 45-year-old laborer with painful primary osteoarthritis
3- A 51-year-old patient with polyarticular rheumatoid arthritis and multiple joint involvement
4- A 71-year-old woman with stage IV rheumatoid arthritis of the elbow
5- A 71-year-old patient with painful radiocapitellar arthritis from rheumatoid arthritis
PREFERRED RESPONSE: 1
DISCUSSION: Distraction interposition arthroplasty is indicated for the treatment of both rheumatoid and posttraumatic arthritis and is reserved for younger patients who are not suitable candidates for total elbow arthroplasty. Although less reliable than prosthetic replacement, distraction interposition arthroplasty is a useful option in the treatment of young, high-demand patients with elbow arthritis. It is rarely indicated in the presence of polyarticular inflammatory arthritis but may be of value in those patients in whom the disease is limited primarily to the elbow. Isolated radiocapitellar arthritis can be successfully treated with radial head resection, although caution should be exercised if there is evidence of instability. Osteoarthritis is best treated with ulnohumeral arthroplasty.
REFERENCES: Cheng SL, Morrey BF: Treatment of the mobile, painful arthritic elbow by distraction interposition arthroplasty. J Bone Joint Surg Br 2000;82:233-238.
Mansat P: Surgical treatment of the rheumatoid elbow. Joint Bone Spine 2001;68:198-210.
24. A 57-year-old man with type I diabetes mellitus has had a tender, erythematous right sternoclavicular joint for the past 2 weeks. Radiographs reveal mild osteolysis without arthritic changes, within normal limits. Management should consist of
1- MRI.
2- sternoclavicular joint aspiration.
3- incision and drainage.
4- IV antibiotics.
5- nonsteroidal anti-inflammatory drugs.
PREFERRED RESPONSE: 2
DISCUSSION: Sternoclavicular joint sepsis is a rare condition that is most often restricted to patients who are immunocompromised, diabetic, or IV drug abusers. Examination commonly reveals a tender, painful, and possibly swollen sternoclavicular joint. If suspicion remains high following a thorough history, physical examination, radiographs, and routine blood tests, joint aspiration should be performed prior to incision and drainage or administration of antibiotics.
REFERENCES: Bremner RA: Monarticular noninfected subacute arthritis of the sternoclavicular joint. J Bone Joint Surg Br 1959;41:749-753.
Rockwood CA Jr: Disorders of the sternoclavicular joint, in Rockwood CA Jr, Matsen FA III (eds): The Shoulder. Philadelphia, PA, WB Saunders, 1998, vol 1, p 488.
25. A 58-year-old man has persistent pain and weakness of his right shoulder after undergoing primary rotator cuff repair 1 year ago. A clinical photograph is shown in Figure 11. Which of the following factors might make functional improvement problematic with revision rotator cuff surgery?
1- Patient’s age
2- Patient’s gender
3- Number of prior surgical procedures
4- Detachment of the deltoid
5- Duration of the rotator cuff tear
PREFERRED RESPONSE: 4
DISCUSSION: Functional improvement after revision rotator cuff surgery is most likely to occur in patients with an intact deltoid, good-quality rotator cuff tissue, preoperative active elevation alone to 90 degrees, and only one prior rotator cuff repair. In this patient, the compromised deltoid origin might make functional improvement less likely.
REFERENCES: Djurasovic M, Marra G, Arroyo JS, et al: Revision rotator cuff repair: Factors influencing results. J Bone Joint Surg Am 2001;83:1849-1855.
Bigliani LU, Cordasco FA, McIlveen SJ, et al: Operative treatment of failed repairs of the rotator cuff. J Bone Joint Surg Am 1992;74:1505-1515.
Neviaser RJ, Neviaser TJ: Operation for failed rotator cuff repair: Analysis of fifty cases.
J Shoulder Elbow Surg 1992;1:283-286.
26. A 31-year-old man sustained a closed injury to his arm in a motor vehicle accident 16 months ago. Treatment of the fracture consisted of intramedullary nailing of the humerus. He now reports pain with minimal activities. Clinical examination and laboratory studies suggest no signs of infection. Radiographs are seen in Figures 12a through 12c. Treatment should now consist of
1- open reduction and exchange humeral nailing.
2- a custom orthosis and an external bone stimulator.
3- removal of the humeral nail and external fixation.
4- removal of the humeral nail and plate fixation with bone grafting.
5- open bone grafting and an internal bone stimulator.
PREFERRED RESPONSE: 4
DISCUSSION: The use of locked nailing for the treatment of established nonunion of the humerus has produced poor results. Since humeral nailing has already failed, exchange humeral nailing without bone grafting has an even less change of success. To increase the likelihood of achieving bony union, the treatment of choice is removal of the humeral nail, dynamic compression plating, and bone grafting.
REFERENCES: Zuckerman J, Giordanno C, Rosen H: Treatment of humeral shaft non-unions, in Bigliani L (ed): Complications of shoulder surgery. Baltimore, MD, William & Wilkins, 1993, pp 173-190.
Jupiter JB: Complex non-union of the humeral diaphysis: Treatment with a medial approach,
an anterior plate, and a vascularized fibular graft. J Bone Joint Surg Am 1990;72:701-707.
27. A well-developed college football player reports swelling and a heaviness in the arm after lifting weights. Examination reveals that distal pulses are normal and equal in both arms. A venogram is shown in Figure 13. What is the most likely cause of this condition?
1- Intimal tearing of the subclavian artery
2- Compression of the subclavian vein by scalene muscle hypertrophy
3- Postural compression of the neurovascular bundle between the clavicle and the first rib
4- A congenital cervical rib
5- Arterial thrombosis induced by repeated overhead activities
PREFERRED RESPONSE: 2
DISCUSSION: The clinical findings indicate venous obstruction without arterial compression, and the venogram reveals occlusion of the subclavian vein, which is most likely the result of thoracic outlet compression. In the developed athlete, scalene muscle hypertrophy (Paget-Schroetter syndrome) causes compression of the subclavian vein. Treatment should consist of thrombolysis followed by decompressive surgery.
REFERENCES: Angle N, Gelabert HA, Farooq MM, et al: Safety and efficacy of early surgical decompression of the thoracic outlet for Paget-Schroetter syndrome. Ann Vasc Surg 2001;15:37-42.
Azakie A, McElhinney DB, Thompson RW, et al: Surgical management of subclavian-vein effort thrombosis as a result of thoracic outlet compression. J Vasc Surg 1998;28:777-786.
28. Figure 14 shows the AP radiograph of a patient who underwent prosthetic arthroplasty 8 years ago and has now become symptomatic again over the past 18 months. A WBC count and erythrocyte sedimentation rate are within normal limits, and aspiration of the glenohumeral joint yields a negative Gram stain and cultures. Which of the following procedures will most likely provide the best pain relief and function?
1- Removal of the humeral and glenoid components
2- Removal of the glenoid component
3- Removal of the loose glenoid component and reimplantation of a new glenoid component
4- Removal of the glenoid component followed by biologic resurfacing of the glenoid
5- Arthrodesis after removal of both components
PREFERRED RESPONSE: 3
DISCUSSION: Simple removal of the loose glenoid component or removal of the loose component followed by implantation of a new glenoid component are both appropriate treatment choices, depending on the remaining glenoid bone stock. However, removal and reimplantation appears to provide the most predictable pain relief and better function than removal alone.
REFERENCES: Antuna SA, Sperling JW, Cofield RH, et al: Glenoid revision surgery after total shoulder arthroplasty. J Shoulder Elbow Surg 2001;10:217-224.
Rodosky MW, Bigliani LU: Surgical treatment of non-constrained glenoid component failure. Oper Tech Orth 1994;4:226-236.
29. A 52-year-old woman reports the sudden onset of intense pain in the right shoulder. She denies any history of injury or previous shoulder problems. At a 2-week follow-up examination, she notes that the pain has decreased, but she now has severe weakness of the external rotators and abductors. Her cervical spine and remaining shoulder examination are otherwise unremarkable. Radiographs of the shoulder and neck are normal. What is the most likely diagnosis?
1- Calcific tendinitis
2- Rotator cuff tendinosis
3- Bursitis
4- Brachial neuritis
5- Glenohumeral arthritis
PREFERRED RESPONSE: 4
DISCUSSION: Patients with brachial neuritis or Parsonage-Turner syndrome usually report the sudden onset of intense pain that subsides in 1 to 2 weeks, followed by weakness for a period of up to 1 year in the muscle that is supplied by the involved nerve. Calcific tendinitis usually can be diagnosed radiographically, with calcium deposits seen in the rotator cuff. Bursitis and rotator cuff tendinosis usually are seen after an increase in activity, and both decrease with rest and medication. Glenohumeral arthritis is a slow, progressive problem that results in a loss of range of motion.
REFERENCES: Misamore GW, Lehman DE: Parsonage-Turner syndrome (acute brachial neuritis). J Bone Joint Surg Am 1996;78:1405-1408.
Dillin L, Hoaglund FT, Scheck M: Brachial neuritis. J Bone Joint Surg Am 1985;67:878-880.
30. What is the most commonly reported complication following elbow arthroscopy?
1- Synovial cutaneous fistula
2- Nerve transection
3- Compartment syndrome
4- Infection
5- Transient neurapraxia
PREFERRED RESPONSE: 5
DISCUSSION: The complication rate following elbow arthroscopy is reported at 5%. The most commonly reported complication is transient neurapraxia, with nerve transection remaining an unfortunate and rare event. While infection remains the most common serious complication, it is uncommon (0.8%). Synovial cutaneous fistula and compartment syndrome, while reported, are the least frequent complications of elbow arthroscopy.
REFERENCES: Kelly EW, Morrey BF, O’Driscoll SW: Complications of elbow arthroscopy.
J Bone Joint Surg Am 2001;83:25-34.
Morrey BF: Elbow complication, in Morrey BF (ed): The Elbow and Its Disorders, ed 3. Philadelphia, PA, WB Saunders, 2001, pp 519-522.
31. When an acute infection of a total elbow arthroplasty is managed with irrigation and debridement, which of the following organisms is associated with the highest risk of persistent infection?
1- Enterococcus cloacae
2- Klebsiella
3- Staphylococcus aureus
4- Staphylococcus epidermidis
5- Streptococcus
PREFERRED RESPONSE: 4
DISCUSSION: Salvage of a total elbow arthroplasty is possible with early aggressive management of acute infection (symptoms for less than 30 days) with serial irrigation and debridement and antibiotic bead placement. This form of treatment is indicated when there are no radiographic or intraoperative signs of loosening. However, successful treatment is largely dependent on the organism. Staphylococcus epidermidis is associated with persistent infection because it is an encapsulating organism, and it is best treated with implant removal and
IV antibiotics.
REFERENCES: Yamaguchi K, Adams RA, Morrey BF: Infection after total elbow arthroplasty. J Bone Joint Surg Am 1998;80:481-491.
Schoifet SD, Morrey BF: Treatment of infection after total knee arthroplasty by debridement with retention of the components. J Bone Joint Surg Am 1990;72:1383-1390.
32. A professional baseball player has had intermittent, mild shoulder pain for the past 2 years. Nonsurgical management has consisted of anti-inflammatory drugs. Examination reveals atrophy of the infraspinatus muscle but not the supraspinatus. There is weakness in external rotation with the arm at his side but not at 90 degrees of abduction. He has no weakness or pain with resisted abduction. Electromyography confirms an isolated lesion of the suprascapular nerve branch to the infraspinatus. He is otherwise neurologically intact. An MRI scan of the shoulder shows no cysts but confirms atrophy of the infraspinatus muscle. What is the next most appropriate step in management?
1- Immediate MRI of the brain
2- Physical therapy and observation
3- Subacromial injection
4- Decompression of the suprascapular nerve at the suprascapular notch
5- Decompression of the infraspinatus branch of the suprascapular nerve at the spinoglenoid notch
PREFERRED RESPONSE: 2
DISCUSSION: Suprascapular nerve injuries are more commonly seen in athletes who participate in overhead activities. When a patient is evaluated for posterior shoulder pain and infraspinatus muscle weakness or atrophy, electrodiagnostic studies are an essential part of the evaluation. In addition, imaging studies are indicated to exclude other diagnoses that can mimic a suprascapular nerve injury. Initial management should consist of cessation of the aggravating activity along with an organized shoulder rehabilitation program. If nonsurgical management fails to provide relief within 6 months to 1 year, surgical exploration of the suprascapular nerve should be considered. Release of the spinoglenoid ligament with resultant suprascapular nerve decompression may result in pain relief and a return of normal shoulder function. In this patient, who has a chronic neuropathy and mild symptoms, surgery is indicated only if nonsurgical management fails to provide relief and he is unable to perform at his position.
REFERENCES: Cummins CA, Bowen M, Anderson K, et al: Suprascapular nerve entrapment at the spinoglenoid notch in a professional baseball pitcher. Am J Sports Med 1999;27:810-812.
Cummins CA, Messer TM, Nuber GW: Suprascapular nerve entrapment. J Bone Joint Surg Am 2000;82:415-424.
Ferretti A, De Carli A, Fontana M: Injury of the suprascapular nerve at the spinoglenoid notch: The natural history of infraspinatus atrophy in volleyball players. Am J Sports Med 1998;26:759-763.
33. A 66-year-old woman who requires a cane for ambulation now notes increasing difficulty in using the cane after undergoing total elbow arthroplasty 3 months ago. AP and lateral radiographs are shown in Figures 15a and 15b. What is the most likely diagnosis?
1- Ulnar nerve neuritis
2- Triceps insufficiency
3- Aseptic loosening
4- Instability
5- Infection
PREFERRED RESPONSE: 2
DISCUSSION: The lateral radiograph reveals a triceps avulsion with a small portion of bone. Triceps weakness and insufficiency can be a symptomatic problem after total elbow arthroplasty and is probably underreported. Ulnar nerve neuritis, aseptic loosening, instability, and infection are all complications of total elbow arthroplasty but would not account for the radiographic findings.
REFERENCES: Koval K (ed): Orthopaedic Knowledge Update 7. Rosemont, IL, American Academy of Orhthopaedic Surgeons, 2002, pp 323-327.
Morrey BF (ed): The Elbow and Its Disorders, ed 3. Philadelphia, PA, WB Saunders, 2000,
pp 598-601.
34. The mother of a healthy 8-month-old boy reports that her son refuses to use his left arm. Examination reveals that the arm hangs limp at his side in an adducted and internally rotated position, and the affected shoulder subluxates posteriorly. Passive external rotation measures 15 degrees. Management should consist of
1- release of the latissimus dorsi and teres major.
2- release of the subscapularis and pectoralis major.
3- passive range-of-motion exercises.
4- exploration of the brachial plexus.
5- functional bracing.
PREFERRED RESPONSE: 3
DISCUSSION: Injury to the upper trunk of the brachial plexus during birth (Erb’s palsy) occurs in approximately 1 in 3,000 births. In a complete lesion, paralysis of the deltoid, supraspinatus, infraspinatus, teres minor, biceps, and brachioradialis results in the findings described above. Spontaneous recovery may occur for up to 2 years. Passive exercises administered daily by the parents are the initial recommended treatment at this age. If significant contracture results in posterior dislocation, surgical correction may be considered.
REFERENCES: Neer CS: Shoulder Reconstruction. Philadelphia, PA, WB Saunders, 1990,
pp 452-454.
Pearl ML: Arthroscopy release of shoulder contracture secondary to birth palsy: An early report on findings and surgical technique. Arthroscopy 2003;19:577-582.
Pearl ML, Edgerton BW, Kon DS, et al: Comparison of arthroscopic findings with MRI and arthrography in children with GH deformity secondary to brachial plexus birth palsy. J Bone Joint Surg Am 2003;85:890-898.
35. During shoulder motion with the elbow controlled in a brace, electromyographic studies of the supraspinatus show significant activity with all range-of-motion testing. Concurrent electromyographic studies of the long head of the biceps will most
likely show
1- minimal activity.
2- moderate activity compared to the supraspinatus.
3- significant activity with a supraspinatus tear.
4- significant activity with an intact supraspinatus.
5- significant activity with both intact and torn rotator cuffs.
PREFERRED RESPONSE: 1
DISCUSSION: During electromyographic studies, the long head of the biceps has been shown to have little activity throughout a wide range of shoulder motion as long as the elbow is immobilized. The supraspinatus is active throughout the range of shoulder motion. Rotator cuff tears do not influence biceps activity as long as the elbow is controlled.
REFERENCES: Yamaguchi K, Riew KD, Galutz LM, et al: Biceps activity during shoulder motion: An electromyographic analysis. Clin Orthop 1997;336:122-129.
Levy AS, Kelly BT, Lintner SA, et al: Function of the long head of the biceps at the shoulder: Electromyographic analysis. J Shoulder Elbow Surg 2001;10:250-255.
36. A right-handed 24-year-old professional baseball player injured his left shoulder 6 weeks ago when he dove forward and landed hard with the arm extended. He reports that the shoulder “slipped out” and “went back in.” The shoulder did not need to be reduced. He now reports deep pain in the front of the shoulder when batting on either side and is hesitant to raise his left arm up over his head to catch a ball. Examination reveals no obvious deformities of the shoulder and a somewhat guarded, limited range of motion in all planes. Provocative tests for the rotator cuff and labrum are equivocal. MRI scans are shown in Figures 16a and 16b. What is the best course of action?
1- Physical therapy
2- Arthroscopic labral repair
3- Arthroscopic subscapularis repair
4- Arthroscopic thermal capsular shift
5- Open Bankart repair
PREFERRED RESPONSE: 1
DISCUSSION: A hard fall on an outstretched arm often results in injury to the glenoid labrum. A significant tear of the anterior/inferior labrum often leads to instability, pain, and mechanical symptoms of the shoulder. The MRI scan shows no obvious labral tear or Hill-Sachs lesion to suggest an anterior dislocation. Recent clinical studies have suggested that early stabilization of initial anterior dislocations may lead to better results than nonsurgical management in young, athletic patients. However, there are no data to support early surgery for anterior labral tears resulting from traumatic subluxation without dislocation. Initial treatment should consist of a short period of rest and immobilization, followed by a physical therapy rehabilitation program designed to restore motion, strength, and dynamic stability to the shoulder. If the athlete cannot return to play following nonsurgical management, surgical repair of the labrum, either through an open or arthroscopic approach, is indicated. There is no role for immediate thermal capsular shift in this setting.
REFERENCES: Abrams JS, Savoie FH III, Tauro JC, et al: Recent advances in the evaluation and treatment of shoulder instability: Anterior, posterior and multidirectional. Arthroscopy 2002;18:1-13.
DeBerardino TM, Arciero RA, Taylor DC, et al: Prospective evaluation of arthroscopic stabilization of acute, initial anterior shoulder dislocations in young athletes: Two- to five-year follow-up. Am J Sports Med 2001;29:586-592.
37. A 35-year-old woman dislocated her right shoulder in a fall from a step stool several months ago. She now reports several painful recurrences. Examination reveals anterior and inferior apprehension that reproduces her symptoms. An MRI scan is shown in Figure 17. Management should consist of
1- open repair of the lateral joint capsule disruption.
2- open Bankart-type repair of the labral-glenoid tear.
3- arthroscopic repair of the Bankart lesion.
4- physical therapy for rotator cuff strengthening.
5- immobilization for 6 weeks, followed by rehabilitation.
PREFERRED RESPONSE: 1
DISCUSSION: The MRI findings reveal a disruption of the humeral insertion of the glenohumeral ligaments and joint capsule (humeral avulsion of the glenohumeral ligament). This lesion has been reported to account for an 8% rate of recurrent dislocation in a subset of patients who are typically older than those with the more common lesions of the glenoid labrum (Bankart lesion). Open repairs have been reported to be successful in the prevention of recurrent instability. Since there is no Bankart lesion, open or arthroscopic labral repairs are not indicated. Nonsurgical management is possible if the patient does not want to undergo surgery; however, the recurrence rate is very high.
REFERENCES: Wolf EM, Cheng JC, Dickson K: Humeral avulsion of glenohumeral ligaments as a cause of anterior shoulder instability. Arthroscopy 1995;11:600-607.
Bokor DJ. Conboy VB. Olson C: Anterior instability of the glenohumeral joint with humeral avulsion of the glenohumeral ligament: A review of 41 cases. J Bone Joint Surg Br
1999;81:93-96.
Coates MH, Breidahl W: Humeral avulsion of the anterior band of the inferior glenohumeral ligament with associated subscapularis bony avulsion in skeletally immature patients. Skeletal Radiol 2001;30:661-666.
38. A 20-year-old man sustained an injury to his arm during a tug-of-war contest. An MRI scan is shown in Figure 18. What is the most likely diagnosis?
1- Lipoma
2- Proximal biceps rupture
3- Distal biceps rupture
4- Biceps and brachialis rupture
5- Biceps brachii transection
PREFERRED RESPONSE: 5
DISCUSSION: The MRI scan reveals a transection of the biceps muscle. The underlying brachialis is intact. This injury can occur as a result of a cord wrapped around the upper arm. Care should be taken to ensure that there is no concurrent vascular injury. A posterior subcutaneous lipoma appears as a well-encapsulated mass on T2-weighted images.
REFERENCES: Heckman JD, Levine MI: Traumatic closed transection of the biceps brachii in the military parachutist. J Bone Joint Surg Am 1978;60:369-372.
Mellen PF: Parachute static line injury with vascular compromise. Mil Med 1989;154:364-365.
39. When the elbow is extended and an axial load is applied, what percent of stress distribution occurs across the ulnohumeral and radiohumeral articular surface, respectively?
1- 20% and 80%
2- 40% and 60%
3- 50% and 50%
4- 60% and 40%
5- 80% and 20%
PREFERRED RESPONSE: 2
DISCUSSION: When load is applied to the wrist, most of the stress is absorbed by the radius. As the load is transferred through the forearm, the interosseous membrane transfers some of the load from the radius to the ulna. The load at the elbow is distributed with 40% at the ulnohumeral articulation and 60% at the radiohumeral articulation.
REFERENCES: Halls AA, Travill R: Transmission of pressure across the elbow joint. Anat Rec 1964;150:243.
Morrey BF (ed): The Elbow and Its Disorders, ed 3. Philadelphia, PA, WB Saunders, 2000.
40. Which of the following is most frequently associated with heterotopic ossification about the shoulder?
1- Acute anterior dislocation
2- Acute posterior dislocation
3- Multiple attempts at closed reduction of chronic unreduced fracture-dislocations
4- Immediate open reduction and internal fixation of greater tuberosity fractures
5- Open reduction of midshaft clavicular fractures
PREFERRED RESPONSE: 3
DISCUSSION: Multiple attempts at closed reduction, delayed surgery for proximal humeral fractures, and associated closed head injury all have been associated with a higher incidence of heterotopic ossification.
REFERENCES: Rockwood CA Jr, Matsen FA III (eds): The Shoulder. Philadelphia, PA, WB Saunders, 1998, vol 1, p 291.
Neer CS: Displaced proximal humerus fractures: Part II. Treatment of three-part and four-part displacement. J Bone Joint Surg Am 1970;52:1090-1103.
41. A 55-year-old woman with a 15-year history of systemic lupus erythematosus has had left shoulder pain for the past 3 months. She reports that the pain has grown progressively worse over the past few months, and her shoulder function is severely limited. She is presently being treated with azathioprine and has used corticosteroids in the past. AP and axillary radiographs are shown in Figures 19a and 19b, and MRI scans are shown in Figures 19c and 19d. Which of the following forms of management will yield the most predictable pain relief and return of shoulder function?
1- Stretching exercises with a physical therapist
2- Arthroscopic debridement
3- Core decompression of the humeral head
4- Humeral hemiarthroplasty
5- Resection of the proximal humerus
PREFERRED RESPONSE: 4
DISCUSSION: Prosthetic shoulder arthroplasty has been shown to provide predictable results for treating stage III and stage IV osteonecrosis of the humeral head. The decision to resurface the glenoid (total shoulder arthroplasty versus humeral hemiarthroplasty) usually is made based on the radiographic and intraoperative appearance of the glenoid. Core decompression of the humeral head has been reported to be effective for earlier stages (pre collapse) but would not be appropriate for a patient with stage IV disease.
REFERENCES: Hattrup SJ, Cofield RH: Osteonecrosis of the humeral head: Results of replacement. J Shoulder Elbow Surg 2000;9:177-182.
L’Insalata JC, Pagnani MJ, Warren RF, et al: Humeral head osteonecrosis: Clinical course and radiographic predictors of outcome. J Shoulder Elbow Surg 1996;5:355-361.
Cruess RL: Steroid-induced avascular necrosis of the head of the humerus: Natural history and management. J Bone Joint Surg Br 1976;58:313-317.
42. A 45-year-old man sustains an acute closed posterolateral elbow dislocation. The elbow is reduced, and examination reveals that the elbow dislocates posteriorly at 35 degrees with the forearm placed in supination. What is the best course of action?
1- Cast immobilization for 6 weeks
2- Hinged brace with early range of motion in supination
3- Hinged brace with early range of motion in pronation
4- Primary ligament repair
5- Lateral collateral ligament reconstruction with tendon graft
PREFERRED RESPONSE: 3
DISCUSSION: Most closed simple dislocations are best managed with early range of motion. Posterior dislocation typically occurs through a posterolateral rotatory mechanism. When placed in pronation, the elbow has greater stability when the medial ligamentous structures are intact. In traumatic dislocations, MRI rarely provides additional information that will affect treatment. In elbows that remain unstable, primary repair is preferred over ligament reconstruction. Cast immobilization increases the risk of arthrofibrosis.
REFERENCE: O’Driscoll SW, Morrey BF, Korinek S, et al: Elbow subluxation and dislocation: A spectrum of instability. Clin Orthop 1992;280:186-197.
43. Figure 20 shows the MRI scan of a 20-year-old athlete who has a painful shoulder. This pathology is most commonly seen in
1- baseball pitchers.
2- downhill skiers.
3- football linemen.
4- volleyball players.
5- tennis players.
PREFERRED RESPONSE: 3
DISCUSSION: The MRI scan reveals a posterior labral detachment. This injury is the result of a posteriorly directed force and is common to football players in blocking positions. Although this injury can occur with trauma in all types of athletes, it is seen with relative frequency in football. Treatment is aimed at labral repair with posterior capsulorrhaphy. Both open and arthroscopic techniques can be used.
REFERENCES: Misamore GW, Facibene WA: Posterior capsulorrhaphy for the treatment of traumatic recurrent posterior subluxations of the shoulder in athletes. J Shoulder Elbow Surg 2000;9:403-408.
Mair SD, Zarzour RH, Speer KP: Posterior labral injury in contact athletes. Am J Sports Med 1998;26:753-758.
44. Figure 21 shows the radiograph of an 18-year-old man who was brought to the emergency department with shoulder pain following a rollover accident on an all-terrain vehicle. Examination reveals a fracture with massive swelling; however, the skin is intact and not tented over the fracture. Based on these findings, initial management should consist of
1- closed reduction of the displaced clavicular fracture.
2- a figure-of-8 clavicular brace to stabilize the clavicular fracture.
3- arteriography to evaluate for vascular injury.
4- electromyography to evaluate for a brachial plexus injury.
5- CT to evaluate for a scapular fracture.
PREFERRED RESPONSE: 3
DISCUSSION: The radiographic and clinical findings suggest a scapulothoracic dissociation with a widely displaced clavicular fracture and a laterally displaced scapula. These injuries have a high association with neurovascular injuries to the brachial plexus and subclavian artery. Emergent vascular evaluation with arteriography and possible vascular repair are indicated. This repair can be combined with open reduction and internal fixation of the clavicle to improve stability. Delay in treatment of these vascular injuries can be devastating.
REFERENCES: Iannotti JP, Williams GR (eds): Disorders of the Shoulder. Philadelphia, PA, Lippincott, 1999, pp 632-635.
Ebraheim NA, An HS, Jackson WT, et al: Scapulothoracic dissociation. J Bone Joint Surg Am 1988;70:428-432.
45. A 55-year-old man reports increasing weakness in his arms that has progressed to his lower limbs, resulting in frequent tripping and falling. Examination reveals weakness in shoulder abduction and external and internal rotation bilaterally. Fasciculation is noted. He also has weakness in elbow flexion and extension bilaterally, and his grip strength is diminished. An electromyogram and nerve conduction velocity studies show decreased amplitude of compound motor action potential, slightly slowed motor conduction velocity, and denervation signs with decreased recruitment in all extremities. The sensory study is normal. Based on these findings, what is the most likely diagnosis?
1- Syrinx of the cervical spine
2- Intracranial mass
3- Myasthenia gravis
4- Amyotrophic lateral sclerosis (ALS)
5- Fascioscapulohumeral dystrophy
PREFERRED RESPONSE: 4
DISCUSSION: The major determinant of ALS (Lou Gehrig disease) is progressive loss of motor neurons. The loss usually begins in one area, is asymmetrical, and later becomes evident in other areas. The first signs of ALS may include either upper or lower motor neuron loss. Recognition of upper motor neuron involvement depends on clinical signs, but electromyography and nerve conduction velocity studies can help identify lower motor neuron involvement. Electrodiagnostic abnormalities in three or more areas are required to make a definitive diagnosis. The motor unit potentials (MUPs) changes in ALS include impaired MUPs recruitment, unstable MUPs, and abnormal MUPs size and configuration. A number of abnormal spontaneous discharges can occur with ALS, especially fibrillation potentials and fasciculation potentials. In ALS, the motor nerve conduction study will be abnormal, but a co-existing normal sensory study is definitive for this disease.
REFERENCES: de Carvalho M, Johnsen B, Fuglsang-Frederiksen A: Medical technology assessment: Electrodiagnosis in motor neuron diseases and amyotrophic lateral sclerosis. Neurophysiol Clin 2001;31:341-348.
Daube JR: Electrodiagnostic studies in amyotrophic lateral sclerosis and other motor neuron disorders. Muscle Nerve 2000;23:1488-1502.
Troger M, Dengler R: The role of electromyography (EMG) in the diagnosis of ALS. Amyotroph Lateral Scler Other Motor Neuron Disord 2000;1:S33-S40.
46. Which of the following positions of immobilization has been shown to best approximate the anterior labrum against the glenoid rim following anterior dislocation of the shoulder?
1- Abduction and external rotation
2- Abduction and internal rotation
3- Adduction and external rotation
4- Adduction and internal rotation
5- Extension
PREFERRED RESPONSE: 3
DISCUSSION: Following anterior dislocation of the shoulder, the affected arm is typically placed in a sling with the shoulder in adduction and internal rotation. A recent study has shown that placement in this position actually results in laxity of the anterior supporting structures of the shoulder, allowing the postinjury hemarthrosis to push the labrum and capsular ligaments away from the anterior glenoid rim. Thus, immobilization in this position may actually impede healing of these structures. Alternatively, resting the arm in a position of adduction and external rotation allows the anterior supporting structures to abut against the anterior glenoid rim by forcing the hemarthrosis posteriorly. Placing the arm in this position following anterior dislocation is believed to allow for better healing of the anterior labrum and ligaments.
REFERENCE: Itoi E, Sashi R, Minagawa H, et al: Position of immobilization after dislocation of the glenohumeral joint: A study with use of magnetic resonance imaging. J Bone Joint Surg Am 2002;84:873-874.
47. What is the most common complication following arthroscopic capsular release in a patient with adhesive capsulitis of the shoulder?
1- Wound infection
2- Failure to maintain range of motion
3- Instability
4- Axillary nerve injury
5- Impingement syndrome
PREFERRED RESPONSE: 2
DISCUSSION: Although all of the above are potential complications after arthroscopic capsular release for adhesive capsulitis, the most common problem is the failure to regain normal glenohumeral motion. An immediate physical therapy program is critical to prevent this complication.
REFERENCES: Ghalambor N, Warner JJP: Arthroscopic capsular release: Evolution of the technique and its applications. Tech Shoulder Elbow Surg 2000;1:52-60.
Pollock RG, Duralde XA, Flatow EL, Bigliani LU: The use of arthroscopy in the treatment of resistant frozen shoulder. Clin Orthop 1994;304:30-36.
48. What is the most common complication following surgical fixation of a distal
humeral fracture?
1- Wound infection
2- Ulnar nerve injury
3- Failure of fixation
4- Nonunion
5- Elbow stiffness
PREFERRED RESPONSE: 5
DISCUSSION: In most series, elbow stiffness is the most common complication and can be overcome by achieving stable fixation and initiating early motion after surgery. All of the other complications are seen but to a lesser degree than elbow stiffness.
REFERENCES: Sanders RA, Raney EM, Pipkin S: Operative treatment of bicondylar intra-articular fractures of the distal humerus. Orthopedics 1992;15:159-163.
Norris TR (ed): Orthopaedic Knowledge Update: Shoulder and Elbow. Rosemont, IL, American Academy of Orthopaedic Surgeons, 1997, pp 397-404.
49. A 16-year-old female swimmer reports several episodes of atraumatic glenohumeral instability that occur with different arm positions. Examination reveals generalized ligamentous laxity and a positive sulcus sign, and her shoulder can be subluxated both anteriorly and posteriorly. Initial management should consist of
1- a strengthening program for the rotator cuff and scapular muscles.
2- arthroscopic thermal capsulorrhaphy.
3- an inferior capsular shift.
4- a glenoid osteotomy.
5- a Putti-Platt repair.
PREFERRED RESPONSE: 1
DISCUSSION: The patient has multidirectional instability (MDI). It has been reported that a high percentage of patients with MDI respond to a properly structured exercise program that is continued for at least 3 to 6 months. If nonsurgical management fails to provide relief, stabilization with an inferior capsular shift procedure has been effective in a high percentage of patients. Unidirectional repairs, such as the Putti-Platt procedure, are unsuitable for correcting MDI. Thermal capsulorrhaphy has been reported to have a very high failure rate
(greater than 50%) for treating MDI.
REFERENCES: Burkhead WZ Jr, Rockwood CA Jr: Treatment of instability of the shoulder with an exercise program. J Bone Joint Surg Am 1992;74:890-896.
Neer CS II, Foster CR: Inferior capsular shift for involuntary inferior and multidirectional instability of the shoulder: A preliminary report. J Bone Joint Surg Am 1980;62:897-908.
Pollock RG, Owens JM, Flatow EL, et al: Operative results of the inferior capsular shift procedure for multidirectional instability of the shoulder. J Bone Joint Surg Am
2000;82:919-928.
Miniaci A, Birnie J: Thermal capsular shrinkage for treatment of multidirectional instability of the shoulder. J Bone Joint Surg Am 2003;85:2283-2287.
50. A 13-year-old boy has a mild deformity of the left sternoclavicular joint after being involved in a rollover accident while riding an all-terrain vehicle. Examination in the emergency department reveals that he is hemodynamically stable, and his neurovascular examination is normal. The CT scan shown in Figure 22 was obtained because radiographs were inconclusive. Management should consist of
1- ice, analgesics, and symptomatic treatment, with delayed reconstruction if necessary.
2- closed reduction in the emergency department.
3- closed reduction in the operating room under anesthesia.
4- open reduction and temporary stabilization with a threaded pin.
5- open reduction and reconstruction of the sternoclavicular joint ligaments.
PREFERRED RESPONSE: 3
DISCUSSION: The CT scan reveals a completely displaced physeal fracture of the medial clavicle with marked posterior displacement of the distal fragment. This fracture pattern is associated with potential injury to the vascular structures of the mediastinum. Reduction should be performed for this fracture and generally can be done closed with shoulder retraction and upward pull on the clavicle with a towel clip. Once reduced, the fracture is relatively stable and typically will heal in good position. Reduction should be performed in the operating room in the event that a vascular injury is detected once compression is removed from the clavicle. Open reduction may be necessary if closed reduction is not possible; however, pinning or ligament reconstruction usually is not necessary.
REFERENCES: Rockwood CA, Matsen FA (eds): The Shoulder, ed 2. Philadelphia, PA,
WB Saunders, 1998, p 581.
Wirth MA, Rockwood CA Jr: Acute and chronic traumatic injuries of the sternoclavicular joint. J Am Acad Orthop Surg 1996;4:268-278.
51. A 42-year-old woman with a long-standing history of rheumatoid arthritis undergoes total shoulder arthroplasty for persistent pain that has failed to respond to nonsurgical management. Intraoperative radiographs reveal an oblique, minimally displaced fracture of the greater tuberosity. Based on these findings, what is the best course of action?
1- Discontinue the procedure and delay completion of the total arthroplasty until the fracture has healed.
2- Cement a long-stemmed humeral component to bypass the fracture site and supplement with cerclage wires.
3- Remove all instrumentation, perform an open reduction and internal fixation of the fracture, and delay completion of the total arthroplasty until the fracture has healed.
4- Insert a standard humeral component and apply a humeral orthosis postoperatively.
5- Insert a standard humeral prosthesis with suture fixation and autogenous cancellous bone grafting of the greater tuberosity fracture.
PREFERRED RESPONSE: 5
DISCUSSION: The risk of intraoperative fracture in osteoporotic bone in patients with rheumatoid arthritis is significant. Fractures most often occur during humeral head dislocation and positioning for canal reaming. If the fracture occurs at the greater tuberosity, cerclage suture fixation of the tuberosity fracture with autogenous cancellous bone graft from the resected humeral head is the treatment of choice.
REFERENCES: Wright TW, Cofield RH: Humeral fractures after shoulder arthroplasty. J Bone Joint Surg Am 1995;77:1340-1346.
Norris TR (ed): Orthopaedic Knowledge Update: Shoulder and Elbow. Rosemont, IL, American Academy of Orthopaedic Surgeons, 1997, pp 215-225.
Frankle MA, Ondrovic LE, Markee BA, et al: Stability of tuberosity reattachment in proximal humeral hemiarthroplasty. J Shoulder Elbow Surg 2002;11:413-420.
52. A 13-year-old gymnast has had recurrent right elbow pain for the past year. She denies any history of trauma. Rest and anti-inflammatory drugs have failed to provide relief. Examination reveals no localized tenderness and only slight loss of both flexion and extension (10 degrees). What is the most likely diagnosis?
1- Recurrent valgus overload (medial collateral ligament sprain)
2- Posterior lateral rotatory instability
3- Biceps tendinitis
4- Medial epicondylitis
5- Osteochondritis of the capitellum
PREFERRED RESPONSE: 5
DISCUSSION: Osteochondritis of the capitellum is characterized by pain, swelling, and limited motion. Catching, clicking, and giving way also can occur. It commonly affects athletes who participate in competitive sports with high stresses, such as pitching or gymnastics.
REFERENCES: Krijnen MR, Lim L, Willems WJ: Arthoscopic treatment of osteochondritis dissecans of the capitellum: Report of 5 female athletes. Arthroscopy 2003;19:210-214.
Schenck RC, Goodnight JM: Osteochondritis dissecans. J Bone Joint Surg Am 1996;3:439-456.
53. The incidence of ipsilateral phrenic nerve blockade after an interscalene
block approaches
1- 15%.
2- 25%.
3- 50%.
4- 75%.
5- 100%.
PREFERRED RESPONSE: 5
DISCUSSION: The most common side effect of an interscalene block is ipsilateral phrenic nerve blockade. The phrenic nerve arises chiefly from the fourth cervical ramus (with contributions from the third and fifth) and is the sole motor supply to the diaphragm. Phrenic nerve palsy usually is well tolerated in healthy patients but should be avoided in patients with limited pulmonary function (severe restrictive or obstructive lung disease, myasthenia gravis, or contralateral hemidiaphragmatic dysfunction). The incidence of ipsilateral phrenic nerve blockade afer interscalene block approaches 100%.
REFERENCES: Long T, Wass C, Burkle C: Perioperative interscalene blockade: An overview of its history and current clinical use. J Clin Anesthesia 2002;14;546-556.
Norris T (ed): Orthopaedic Knowledge Update: Shoulder and Elbow 2. Rosemont, IL, American Academy of Orthopaedic Surgeons, 2002, pp 433-442.
54. What is the most consistent finding regarding glenohumeral kinematics in patients with symptomatic tears of the rotator cuff?
1- No superior translation during abduction
2- Retention of ball-and-socket kinematics with more than 30 degrees of abduction
3- Loss of ball-and-socket kinematics with less than 30 degrees of abduction
4- Superior translation of the humeral head with more than 30 degrees of abduction
5- Superior translation of the humeral head with external rotation
PREFERRED RESPONSE: 4
DISCUSSION: Normal glenohumeral kinematics are represented by ball-and-socket modeling when the rotator cuff is intact. This is true for motion that involves more than 30 degrees of abduction. In patients with shoulder pain and symptomatic rotator cuff tears, superior translation occurs with abduction beyond 30 degrees. This is quite evident in massive tears but is seen consistently to a lesser degree with smaller tears.
REFERENCES: Yamaguchi K, Sher JS, Anderson WK, et al: Glenohumeral motion in patients with rotator cuff tears: A comparison of asymptomatic and symptomatic shoulders. J Shoulder Elbow Surg 2000;9:6-11.
Poppen NK, Walker PS: Normal and abnormal motion of the shoulder. J Bone Joint Surg Am 1976;58:195-201.
55. A 28-year-old man sustained numerous injuries in an accident including a dislocation of the elbow and a severe closed head injury that resulted in unconsciousness. The elbow was reduced in the emergency department. After 1 month of rehabilitation, the patient reports pain and stiffness. A radiograph is shown in Figure 23. Management should now consist of
1- semiconstrained total elbow arthroplasty.
2- ulnohumeral arthroplasty and anterior and posterior capsular releases.
3- closed reduction and external fixation.
4- open reduction, heterotopic bone excision, anterior and posterior capsular releases, and a hinged elbow fixator.
5- open reduction, heterotopic excision, anterior and posterior capsular releases, and pin fixation across the joint for 3 weeks.
PREFERRED RESPONSE: 4
DISCUSSION: In a young individual with a chronic dislocation of the elbow and heterotopic bone formation, the treatment of choice is open reduction, heterotopic bone excision, anterior and posterior capsular releases, and a dynamic hinged fixator to begin protected early postoperative range of motion. It is important to understand that the fixator protects the reconstruction and allows early range of motion, but it does not maintain the reduction and should not be expected to do so. Pin fixation across the elbow delays early motion and is not recommended. Total elbow arthroplasty is not indicated, and ulnohumeral arthroplasty is for a primary arthritic condition.
REFERENCES: Garland DE, Hanscom DA, Keenan MA, et al: Resection of heterotopic ossification in the adult with head trauma. J Bone Joint Surg Am 1985;67:1261-1269.
Moor TJ: Functional outcome following surgical excision of heterotopic ossification in patients with traumatic brain injury. J Orthop Trauma 1993;7:11-14.
56. A 52-year-old man has had right shoulder pain in the deltoid region that increases at night for the past 2 months. He denies any history of trauma. Examination reveals mild tenderness over the greater tuberosity, and the Neer and Hawkins impingement signs are positive. AP and outlet lateral radiographs are shown in Figures 24a and 24b. Initial management should consist of
1- a program of stretching exercises and rotator cuff strengthening exercises.
2- a series of six cortisone injections.
3- arthroscopic acromioplasty.
4- arthroscopic acromioplasty and laser capsulorrhaphy.
5- open acromioplasty.
PREFERRED RESPONSE: 1
DISCUSSION: The patient has the findings of classic subacromial impingement. Initial management should consist of stretching exercises directed at the posterior capsule and a program of rotator cuff and deltoid strengthening exercises performed below the horizontal in a “safe” plane. The judicious use of subacromial cortisone injections (one or two) may be helpful. Anterior acromioplasty is reserved for patients who have failed to respond to nonsurgical management.
REFERENCES: Morrison DS, Frogameni AD, Woodworth P: Non-operative treatment of subacromial impingement syndrome. J Bone Joint Surg Am 1997;79:732-737.
Neer CS: Impingement lesions. Clin Orthop 1983;173:70-77.
Blair B, Rokito AS, Cuomo F, et al: Efficacy of injections of corticosteroids for subacromial impingement syndrome. J Bone Joint Surg Am 1996;78:1685-1689.
57. A 70-year-old woman is brought to the emergency department with a two-part greater tuberosity fracture with an anterior subcoracoid dislocation. One day after successful closed reduction, examination reveals marked swelling of the involved arm, forearm, and hand, as well as large amounts of “weeping” serous fluid but no obvious lacerations. The fingers are warm and pink, and the pulses are normal distally with good refill. Edema is present. There is no pain with passive and active motion of the elbow, wrist, and fingers. What is the next most appropriate step in management?
1- Sympathetic stellate block
2- Emergent fasciotomy
3- Emergent arterial thrombectomy
4- Venous duplex ultrasound studies
5- Arteriography
PREFERRED RESPONSE: 4
DISCUSSION: Although not as common as arterial injury, venous thrombosis secondary to trauma of the subclavian or axillary vein can be problematic; therefore, venous duplex ultrasound scanning is the diagnostic study of choice. Arteriography may not show venous thrombosis in the venous run-off phase. The clinical history does not fit the usual presentation of a compartment syndrome or complex regional pain syndrome.
REFERENCE: Killewich LA, Bedford GR, Black KW, et al: Diagnosis of deep venous thrombosis: A prospective study comparing duplex scanning to contrast venography.
Circulation 1989;79:810.
58. A baseball pitcher has intractable posterior and superior shoulder pain. The arthroscopic view seen in Figure 25 shows no Bankart or Hill-Sachs lesion and a negative drive-through sign. There are no signs of ligamentous laxity, but active compression and anterior slide tests are positive. Treatment should consist of
1- open fixation of the SLAP lesion with a screw that can be removed later.
2- arthroscopic repair of the SLAP lesion with suture anchors.
3- arthroscopic repair of the SLAP lesion with suture anchors with a thermal capsular shift.
4- arthroscopic repair of the SLAP lesion with suture anchors and a rotator cuff interval closure.
5- arthroscopic repair of the SLAP lesion with suture anchors and an arthroscopic capsular placation.
PREFERRED RESPONSE: 2
DISCUSSION: According to Morgan and associates, a type II SLAP lesion can create or is associated with a superior instability pattern. They suggest that this can exist without a co-existing anteroinferior instability pattern. They reported that repair of the SLAP lesion alone resulted in satisfactory outcomes in 90% of patients and a return to throwing in more than 90% of pitchers. The arthroscopic findings in this patient do not support a diagnosis of anteroinferior laxity or instability; therefore, thermal capsular shift or capsular placation is not necessary.
REFERENCES: Morgan CD, Burkhart SS, Palmeri M, et al: Type II SLAP lesions: Three subtypes and their relationships to superior instability and rotator cuff tears. Arthroscopy 1998;14:553-565.
Mileski RA, Snyder RJ: Superior labral lesions in the shoulder: Pathoanatomy and surgical management. J Am Acad Orthop Surg 1998;6:121-131.
Levitz CL, Dugas J, Andrews JR: The use of arthroscopic thermal capsulorrhaphy to treat internal impingement in baseball players. Arthroscopy 2001;17:573-577.
59. With increasing abduction in the scapular plane, maintaining neutral rotation, contact area, and contact pressure per unit area between the humeral head and glenoid follows what pattern if the total load across the joint is held constant?
1- Contact area and contact pressure both decrease.
2- Contact area and contact pressure both increase.
3- Contact area and contact pressure both remain the same.
4- Contact area increases and contact pressure decreases.
5- Contact area decreases and contact pressure increases.
PREFERRED RESPONSE: 4
DISCUSSION: The glenohumeral joint becomes more congruent at higher levels of abduction. As a consequence, contact area increases. As the load is spread more evenly across the joint, contact pressure per unit area decreases as long as the total load across the joint is held constant.
REFERENCES: Warner JJP, Bowen MK, Deng XH, et al: Articular contact patterns of the normal glenohumeral joint. J Shoulder Elbow Surg 1998;7:381-388.
Greis PE, Scuderi MG, Mohr A, et al: Glenohumeral articular contact areas and pressures following labral and osseous injury to the anteroinferior quadrant of the glenoid. J Shoulder Elbow Surg 2002;11:442-451.
60. A 21-year-old patient has had pain and a marked decrease in active and passive shoulder motion after having had a seizure 2 months ago as the result of alcohol abuse. Current AP and axillary radiographs and a CT scan are shown in Figures 26a through 26c. Management should consist of
1- closed reduction under sedation.
2- total shoulder arthroplasty.
3- open reduction and subscapularis and lesser tuberosity transfer.
4- open reduction and disimpaction with bone grafting.
5- hemiarthroplasty with the humeral component placed in less than 20 degrees of retroversion.
PREFERRED RESPONSE: 3
DISCUSSION: Open reduction and subscapularis and lesser tuberosity transfer into the defect is the treatment of choice in young individuals who have defects that involve between 20% to 45% of the head. Disimpaction and bone grafting is an option in injuries that are less than 3 weeks old. Closed reduction 2 to 3 months after injury usually is unsuccessful and increases the risk of fracture or neurovascular injury. Total shoulder arthroplasty is reserved for defects of greater than 50% or with associated glenoid surface damage. Hemiarthroplasty should be avoided in young individuals unless 50% or more of the head is involved.
REFERENCES: Gerber C: Chronic locked anterior and posterior dislocations, in Warner JJ, Iannotti JP, Gerber C (eds): Complex and Revision Problems in Shoulder Surgery. Philadelphia, PA, Lippincott-Raven, 1997, pp 99-113.
Hawkins RJ, Neer CS II, Pianta RM, et al: Locked posterior dislocation of the shoulder. J Bone Joint Surg Am 1987;69:9-18.
61. Which of the following ligaments are the primary static restraints to inferior translation of the arm when the shoulder is in 0 degrees of abduction and neutral rotation?
1- Inferior and middle glenohumeral
2- Middle and superior glenohumeral
3- Superior glenohumeral and coracohumeral
4- Coracohumeral and coracoclavicular
5- Coracoacromial and coracoclavicular
PREFERRED RESPONSE: 3
DISCUSSION: Biomechanical ligament sectioning studies have implicated both the superior glenohumeral and coracohumeral ligaments as restraints to inferior translation when the shoulder is in 0 degrees of abduction and neutral rotation. Although there is controversy over the significance of each ligament, both are involved to some degree. The middle glenohumeral ligament is more important in the midranges of abduction, and the inferior ligament is more important at 90 degrees of abduction. The coracoacromial and coracoclavicular ligaments play no role in glenohumeral restraint.
REFERENCES: Warner JJ, Deng XH, Warren RF, et al: Static capsuloligamentous restraints to superior-inferior translation of the glenohumeral joint. Am J Sports Med 1992;20:675-685.
Jost B, Koch PP, Gerber C: Anatomy and function of the rotator interval. J Shoulder Elbow Surg 2000;9:336-341.
62. A 44-year-old man who sustained an elbow dislocation 3 months ago now reports pain and restricted elbow motion. Radiographs are shown in Figures 27a and 27b. Management should consist of
1- closed reduction and casting.
2- static splinting.
3- open reduction and lateral collateral ligament reconstruction.
4- open reduction and lateral collateral ligament repair.
5- open reduction, application of a hinged external fixator, and radial head arthroplasty.
PREFERRED RESPONSE: 5
DISCUSSION: The treatment of choice for an ankylosed chronically dislocated elbow is surgical reduction. Open reduction with application of an external fixator provides excellent results for this complex problem. Radial head arthroplasty is indicated for a radial head fracture that cannot be reconstructed. Attempts at closed reduction will be unsuccessful and should not be attempted in a stiff elbow. In chronic dislocations, direct reinsertion of injured ligaments is not feasible because of soft-tissue contracture.
REFERENCES: Jupiter J, Ring D: Treatment of unreduced elbow dislocation with hinged external fixation. J Bone Joint Surg Am 2002;84:1630-1635.
Moro JK, Werier J, MacDermid JC, et al: Arthroplasty with a metal radial head for unreconstructible fractures of the radial head. J Bone Joint Surg Am 2001;83:1201-1211.
63. A 67-year-old man who underwent humeral head arthroplasty for a four-part fracture 6 months ago reports that he is still unable to actively elevate his arm. Rehabilitation after surgery consisted of a sling with passive range-of-motion exercises for 2 weeks and then progressed to active-assisted and strengthening exercises at 3 weeks. Radiographs are shown in Figures 28a and 28b. What is the primary cause of his inability to elevate
the arm?
1- Rotator cuff tear
2- inadequate strengthening exercises
3- Instability
4- Tuberosity nonunion
5- Prosthetic loosening
PREFERRED RESPONSE: 4
DISCUSSION: The radiographs show nonunion of both the greater and lesser tuberosities. Tuberosity pull-off and nonunion remain among the most common causes of failed humeral head arthroplasty for fracture. Strict attention to securing the tuberosities to each other and to the shaft, and autogenous bone grafting from the excised humeral head will decrease the incidence of pull-off and improve healing rates. Active-assisted range-of-motion and strengthening exercises should be delayed until tuberosity healing is noted radiographically, usually at 6 to 8 weeks postoperatively.
REFERENCES: Hartsock LA, Estes WJ, Murray CA, et al: Shoulder hemiarthroplasty for proximal humeral fractures. Orthop Clin North Am 1998;29:467-475.
Hughes M, Neer CS: Glenohumeral joint replacement and postoperative rehabilitation.
Phys Ther 1975;55:850-858.
Compito CA, Self EB, Bigliani LU: Arthroplasty and acute shoulder trauma. Clin Orthop 1994;307:27-36.
64. Initial postoperative management after repair of an acute rotator cuff tear includes
1- active range of motion.
2- active abduction to prevent scarring.
3- passive forward elevation and external rotation within a safe zone determined at surgery.
4- eccentric strengthening exercises.
5- internal rotation behind the back.
PREFERRED RESPONSE: 3
DISCUSSION: In the immediate postoperative period following repair of an acute rotator cuff tear, passive forward elevation and external rotation should be performed within the safe zone determined at surgery. Early active range of motion (prior to tendon healing), internal rotation behind the back, and resistive exercises increase the risk of rupture of the repair.
REFERENCES: Iannotti JP: Full-thickness rotator cuff tear: Factors affecting surgical outcome. J Am Acad Orthop Surg 1994;2:87-95.
Bigliani LU, Cordasco FA, McIlveen ST, et al: Operative repair of massive rotator cuff tears: Long-term result. J Shoulder Elbow Surg 1992;1:120-130.
65. A 34-year-old woman reports constant midlateral arm pain after sustaining minimal trauma to the shoulder. Radiographs and a biopsy specimen are shown in Figures 29a and 29b. What is the most likely diagnosis?
1- Chondroblastoma
2- Lymphoma
3- Giant cell tumor
4- Chondromyxoid fibroma
5- Desmoplastic fibroma
PREFERRED RESPONSE: 3
DISCUSSION: Eighty percent of giant cell tumors occur in patients older than age 20 years, with the peak incidence in the third decade of life. Most of these tumors are eccentrically located and epiphyseal in location. They are lytic in nature as in this patient. Although named for the hallmarked multinucleated giant cells seen in the lesion, the basic cell type is the spindle-shaped stromal cell. Chondroblastoma is highly cellular and contains large multinucleated giant cells with intercellular chondroid material, some of which is calcified. Chondromyxoid fibroma has chondroid tissue separated by strands of more cellular tissue with occasional multinucleated giant cells. Desmoplastic fibroma is characterized by poorly cellular fibrous tissue, and lymphoma is highly cellular with characteristic round cells.
REFERENCES: Campanacci M, Baldini N, Boriani S, et al: Giant cell tumor of bone. J Bone Joint Surg Am 1987;69:106-114.
Goldenberg RR, Campbell CJ, Bonfiglio M: Giant cell tumor of bone: An analysis of two hundred and eighteen cases. J Bone Joint Surg Am 1970;52:619-664.
66. A 25-year-old professional baseball pitcher reports a 4-month history of gradually increasing medial elbow pain that occurs during the late cocking and acceleration phases of throwing. The pain occasionally refers distally along the ulnar aspect of the forearm. He denies any weakness; however, he notes occasional paresthesias. A nerve conduction velocity study demonstrates increased latency across the cubital tunnel. Management consisting of 6 weeks of rest and rehabilitation fails to provide relief as the symptoms returned when he resumed throwing. What is the best course of action?
1- Further rehabilitation
2- Corticosteroid injection
3- Reconstruction of the medial collateral ligament
4- Subcutaneous transposition of the ulnar nerve
5- Arthroscopic debridement of medial osteophytes
PREFERRED RESPONSE: 4
DISCUSSION: In the thrower’s elbow, ulnar neuritis is felt to result from both chronic compression and traction on the nerve that occurs during the throwing motion. Occasionally, subluxation of the nerve also can lead to symptoms. If nonsurgical management fails to provide relief, transposition of the nerve to an anterior subcutaneous location is the surgical procedure of choice. The nerve is held in its new position by one or two fascial slings created from the fascia of the common flexor origin.
REFERENCES: Schickendantz MS: Diagnosis and treatment of elbow disorders in the overhead athlete. Hand Clin 2002;18:65-75.
Eaton RG, Crowe JF, Parkes JC III: Anterior transposition of the ulnar nerve using a non-compressing fasciodermal sling. J Bone Joint Surg Am 1980;62:820-825.
67. What artery provides the only direct vascularizaton to both the intraneural and extraneural blood supply of the ulnar nerve just proximal to the cubital tunnel?
1- Superior ulnar collateral
2- Inferior ulnar collateral
3- Posterior ulnar recurrent
4- Brachial
5- Ulnar
PREFERRED RESPONSE: 2
DISCUSSION: The superior ulnar collateral, inferior ulnar collateral, and posterior ulnar recurrent arteries provide consistent vascular supply to the ulnar nerve. This supply is segmental in nature. No identifiable direct anastomosis is seen between the superior ulnar collateral and the posterior ulnar recurrent arteries. The inferior ulnar collateral artery provides the only direct vascularization to the nerve and is located in the region just proximal to the cubital tunnel. The segmental nature of the blood supply to the ulnar nerve underscores the importance of its preservation during transposition.
REFERENCE: Yamaguchi K, Sweet FA, Bindra R, et al: The extraneural and intraneural arterial anatomy of the ulnar nerve at the elbow. J Shoulder Elbow Surg 1999;8:17-21.
68. A 56-year-old man underwent right total shoulder arthroplasty 2 months ago. Recently while reaching with his shoulder in a flexed and adducted position, he noted shoulder pain and afterwards he could not externally rotate his arm. An axillary radiograph is shown in Figure 30. What is the most likely cause of this problem?
1- Excessive anteversion of the humeral component
2- Excessive anteversion of the glenoid component
3- Excessive posterior capsular laxity
4- Anterior capsular laxity
5- Use of an oversized humeral head
PREFERRED RESPONSE: 3
DISCUSSION: Anteversion of the humeral component may result in anterior instability of the component. Posterior instability after total shoulder arthroplasty is usually the result of some combination of the following factors: untreated anterior soft-tissue contractures, excessive posterior capsular laxity, and excessive retroversion of the humeral and/or glenoid components.
REFERENCES: Cofield RH, Edgerton BC: Total shoulder arthroplasty: Complications and revision surgery. Instr Course Lect 1990;39:449-462.
Wirth MA, Rockwood CA Jr: Complications of total shoulder replacement arthroplasty. J Bone Joint Surg Am 1996;78:603-616.
69. A 70-year-old man seen in the emergency department has had left shoulder pain and a fever of 101.5 degrees F (38.6 degrees C) for the past 3 days. He denies any history of trauma. Examination reveals tenderness anterosuperiorly and at the posterior glenohumeral joint line. He has very limited range of motion (passive and active). Laboratory studies show a WBC count of 12,000/mm3 and an erythrocyte sedimentation rate of 48 mm/h. Initial management should consist of
1- an oral cephalosporin antibiotic and discharge home.
2- IV oxacillin and gentamicin.
3- arthroscopic drainage of the glenohumeral joint.
4- open irrigation and drainage of the glenohumeral joint.
5- aspiration of the glenohumeral joint and subacromial space with Gram stain and culture of the fluid.
PREFERRED RESPONSE: 5
DISCUSSION: It appears that the patient has septic arthritis of the glenohumeral joint; therefore, initial management should consist of aspiration of the glenohumeral joint and subacromial space separately, followed by Gram stain and culture of the fluid. Based on the findings, broad-spectrum IV antibiotics should be started. If the diagnosis of septic arthritis is confirmed, then arthroscopic or open surgical drainage usually is indicated.
REFERENCES: Sawyer JR, Esterhai JL Jr: Shoulder infections, in Warner JJ, Iannotti JP, Gerber C (eds): Complex and Revision Problems in Shoulder Surgery. Philadelphia, PA, Lippincott-Raven, 1997.
Leslie BM, Harris JM, Driscoll D: Septic arthritis of the shoulder in adults. J Bone Joint Surg Am 1989;71:1516-1522.
70. A 40-year-old man who is an avid weight lifter has had chronic pain in the proximal anterior shoulder for the past year. He denies any history of trauma. Examination reveals tenderness at the intertubercular groove, a positive speed test, and a positive Neer impingement sign. Nonsurgical management has failed to provide relief, and he is now considering surgery. Arthroscopic findings in the glenohumeral joint are shown in Figure 31. Based on these findings, treatment should consist of
1- debridement of the biceps tendon.
2- debridement of the rotator cuff tear.
3- repair of the biceps pulley system.
4- repair of the rotator cuff tear.
5- tenodesis of the biceps tendon.
PREFERRED RESPONSE: 5
DISCUSSION: The arthroscopic image shows a tear through more than 50% of the biceps tendon; therefore, treatment should consist of tenodesis or tenotomy of the tendon. However, because this patient is relatively young and active, the treatment of choice is tenodesis of the biceps tendon.
REFERENCES: Sethi N, Wright R, Yamaguchi K: Disorders of the long head of the biceps tendon. J Shoulder Elbow Surg 1999;8:644-654.
Eakin CL, Faber KJ, Hawkins RJ, et al: Biceps tendon disorders in athletes. J Am Acad Orthop Surg 1999;7:300-310.
Burkhead WZ, Arcand MA, Zeman C, et al: The biceps tendon, in Rockwood CA, Matsen FA, Wirth MA, et al (eds): The Shoulder, ed 2. Philadelphia, PA, WB Saunders, 1996.
71. A 59-year-old man reports moderate shoulder pain and very restricted range of motion after undergoing humeral arthroplasty for osteoarthritis 1 year ago. An AP radiograph is shown in Figure 32. Management should now consist of
1- an aggressive program of stretching exercises.
2- soft-tissue release and subscapularis lengthening.
3- exchange of the modular humeral head to a smaller size, with glenoid arthroplasty.
4- revision of the humeral component, re-cutting of the humeral neck, soft-tissue releases, and glenoid arthroplasty.
5- glenohumeral arthrodesis.
PREFERRED RESPONSE: 4
DISCUSSION: The radiograph reveals that an insufficient amount of the proximal humerus was excised in the index procedure, resulting in malalignment of the humeral component, overstuffing of the glenohumeral joint, and glenoid arthritis. It is unlikely that physical therapy or soft-tissue releases alone will be adequate. Revision of the humeral component, recutting of the proximal humerus to allow a more anatomic alignment of the humeral component, appropriate soft-tissue releases, and glenoid arthroplasty will offer the best chance of improvement in this difficult situation.
REFERENCES: Neer CS II, Kirby RM: Revision of humeral head and total shoulder arthroplasties. Clin Orthop 1982;170:189-195.
Petersen SA, Hawkins RJ: Revision of failed total shoulder arthroplasty. Orthop Clin North Am 1998;29:519-533.
72. A 70-year-old woman has a preoperative anterior interscalene block prior to undergoing a total shoulder arthroplasty. After seating her in the beach chair position, she becomes acutely hypotensive. What is the most likely cause for the hypotension?
1- Tension pneumothorax
2- Inadvertent epidural injection
3- Inadvertent intravascular injection
4- Laryngeal nerve block
5- Bezold-Jarisch reflex
PREFERRED RESPONSE: 5
DISCUSSION: The beach chair position may cause sudden hypotension and bradycardia as a result of the Bezold-Jarisch reflex. This reflex occurs when venous pooling and increased sympathetic tone induce a low-volume, hypercontractile ventricle, resulting in activation of the parasympathetic nervous system and sympathetic withdrawal. The reported incidence of this phenomenon associated with the sitting position is between 13% to 24%. Left untreated, the result may be cardiac arrest. Pneumothorax or central nervous system toxicity after interscalene block is rare and has an incidence of less than 0.2%. Laryngeal nerve block associated with interscalene nerve block can occur but usually results in hoarseness secondary to ipsilateral vocal cord palsy.
REFERENCES: Long T, Wass C, Burkle C: Perioperative interscalene blockade: An overview of its history and current clinical use. J Clin Anesthesia 2002;14:546-556.
Norris T (ed): Orthopaedic Knowledge Update: Shoulder and Elbow 2. Rosemont, IL, American Academy of Orthopaedic Surgeons, 2002, pp 433-442.
73. What structure is considered the single most important soft-tissue restraint to anterior-posterior stability of the sternoclavicular joint?
1- Posterior capsular ligament
2- Anterior capsular ligament
3- Interclavicular ligament
4- Intra-articular disk
5- Subclavius tendon
PREFERRED RESPONSE: 1
DISCUSSION: In a cadaver ligament sectioning study, the posterior capsular ligament was considered the most important structure for anterior-posterior stability of the sternoclavicular joint. The anterior capsular ligament also helps prevent anterior displacement but not to the same degree as the posterior ligament. The interclavicular ligament provides little support for anteroposterior translation.
REFERENCES: Spencer EE, Kuhn JE, Huston LJ, et al: Ligamentous restraints to anterior and posterior translation of the sternoclavicular joint. J Shoulder Elbow Surg 2002;11:43-47.
Rockwood CA Jr, Matsen FA III, Jobe CM: Gross Anatomy of the Shoulder. Philadelphia, PA, WB Saunders, 1998.
74. A 35-year-old man has atraumatic painless limited elbow motion. Radiographs are shown in Figures 33a and 33b. What is the most likely diagnosis?
1- Melorheostosis
2- Ectopic bone formation
3- Bone infarct
4- Infection
5- Juxacortical chondroma
PREFERRED RESPONSE: 1
DISCUSSION: Based on the radiographic findings, the patient has melorheostosis, a rare, benign connective tissue disorder that is characterized by a cortical thickening of bone. It produces a “dripping candle wax” appearance with dense hyperostosis that flows along the cortex. Ectopic bone formation is a consideration but is associated with injuries or burns. Bone infarcts produce intraosseous sclerosis typically affecting the distal femur with the “smoke up chimney” appearance. Infection is always a consideration but typically does not have the linear osteitis seen in melorheostosis. Juxacortical chondroma is a benign cartilage growth that arises from the capsule and may involve the underlying cortical bone but rarely the medullary canal.
REFERENCES: Campbell CJ, Papademetriou T, Bonfiglio M: Melorheostosis: A report of the clinical, roentgenographic, and pathological findings in fourteen cases. J Bone Joint Surg Am 1968;50:1281-1304.
Kawabata H, Tsuyuguchi Y, Kawai H, et al: Melorheostosis of the upper limb: A report of two cases. J Hand Surg Am 1984;9:871-876.
75. A 64-year-old man who underwent total shoulder arthroplasty 4 weeks ago is making satisfactory progress in physical therapy, but his therapist notes limitations in external rotation to neutral. A stretching program is started, and the patient suddenly gains 90 degrees of external rotation but now reports increased pain and weakness. What is the best course of action?
1- Early surgical exploration and repair of the torn subscapularis tendon
2- Observation and reassurance that the pain will resolve
3- A slow progressive resistance program to restore strength
4- CT to assess for component malrotation
5- Electromyography to evaluate for possible nerve injury
PREFERRED RESPONSE: 1
DISCUSSION: Nearly all approaches to shoulder arthroplasty require detachment of the subscapularis tendon from the humerus and subsequent repair. Healing of this tenotomy is one of the limiting factors in postoperative recovery. Failure of the tenotomy repair must be recognized and treated early with repeat repair or pectoralis muscle transfer for optimal results. Failure of the subscapularis is diagnosed clinically as excessive external rotation and weakness, especially in the lift-off or belly press position. Muscle testing can be difficult in the postoperative period and may not be possible to assess in those positions. Although MRI might be useful to confirm the diagnosis, studies may be limited by artifact. CT or electromyography would not be diagnostic.
REFERENCES: Wirth MA, Rockwood CA Jr: Complications of total shoulder-replacement arthroplasty. J Bone Joint Surg Am 1996;78:603-616.
Miuer SL, Hazrati Y, Klepps S, et al: Loss of subscapularis function after shoulder replacement: A seldom recognized problem. J Shoulder Elbow Surg 2003;12:29-34.
76. A 52-year-old man has shoulder pain and stiffness after undergoing a “mini-lateral” rotator cuff repair 6 months ago. Examination reveals that he is afebrile with normal vital signs. There is slight erythema but no drainage from the incision. Range of motion is limited in all planes, and there is weakness with resisted external rotation and abduction. Radiographs show a well-positioned metal implant within the greater tuberosity. Laboratory studies reveal a WBC count of 8,400/ mm3 (normal 3,500 to 10,500/ mm3) and an erythrocyte sedimentation rate of 63 mm/h (normal up to 20 mm/h). What is the next most appropriate step in management?
1- Subacromial corticosteroid injection
2- Aspiration of the subacromial and glenohumeral joint spaces
3- Nonsteroidal anti-inflammatory drugs
4- Extensive surgical debridement
5- Diagnostic arthroscopy
PREFERRED RESPONSE: 2
DISCUSSION: Deep sepsis of the shoulder following rotator cuff repair is an uncommon problem. Patients with infections of this type typically report persistent pain and are not systemically ill. They may have signs of local wound problems such as erythema, drainage, and dehiscence. Laboratory studies can be helpful in making an accurate diagnosis. Most patients will not show a significant elevation of the WBC count; however, an elevated erythrocyte sedimentation rate is nearly always present and should alert the clinician to the presence of infection. Aspiration of both subacromial and glenohumeral joint spaces is necessary to confirm the diagnosis. The most effective treatment for deep shoulder sepsis following rotator cuff repair involves extensive surgical debridement, removing all suspicious soft tissue as well as implants. Administration of appropriate antibiotic therapy is needed for complete control of the infection.
REFERENCES: Mirzayan R, Itamura JM, Vangsness CT, et al: Management of chronic deep infection following rotator cuff repair. J Bone Joint Surg Am 2000;82:1115-1121.
Settecerri JJ, Pitnu MA, Rock MG, et al: Infection after rotator cuff repair. J Shoulder Elbow Surg 1994;8:105.
Herrera MF, Bauer G, Reynolds F, et al: Infection after mini-open rotator cuff repair. J Shoulder Elbow Surg 2002;11:605-608.
77. A 21-year-old pitcher reports shoulder pain with hard throwing. He notes that the pain occurs in the early acceleration phase of his throw. Given his history, what structures are at greatest risk for injury?
1- Posterosuperior labrum, greater tuberosity, articular side of the rotator cuff
2- Posterior glenoid, humeral head, bursal side of the rotator cuff
3- Biceps anchor, articular side of the rotator cuff, supraspinatus tendon
4- Biceps tendon, bursal side of the rotator cuff, superior labrum
5- Subscapularis, anterior labrum, humeral head
PREFERRED RESPONSE: 1
DISCUSSION: Internal impingement in the thrower’s shoulder occurs in the abducted, externally rotated position as described by Walch and associates. The injury is thought to occur from repetitive contact between the posterosuperior portion of the labrum and glenoid against the articular side of the rotator cuff and greater tuberosity.
REFERENCES: Paley KJ, Jobe FW, Pink MM, et al: Arthroscopic findings in the overhand throwing athlete: Evidence for posterior internal impingement of the rotator cuff. Arthroscopy 2000;16:35-40.
Jazrawi LM, McCluskey GM III, Andrews JR: Superior labral anterior and posterior lesions and internal impingement in the overhead athlete. Instr Course Lect 2003;52:43-63.
Walch G, Boileau P, Noel E, et al: Impingement of the deep surface of the supraspinatus tendon on the posterosuperior glenoid rim: An arthroscopic study. J Shoulder Elbow Surg
1992;1:238-245.
78. A 30-year-old man landed on his shoulder in a fall off his mountain bike. An AP radiograph and CT scan are shown in Figures 34a and 34b. Management should
consist of
1- immobilization in a sling and swathe.
2- open reduction and internal fixation.
3- humeral arthroplasty.
4- subscapularis transfer into the humeral head articular defect.
5- a hanging cast.
PREFERRED RESPONSE: 2
DISCUSSION: The radiograph shows a valgus impacted four-part fracture. The humeral head is deeply depressed into the metaphysis but is still articulating with the glenoid as seen on the CT scan. Unlike a “classic” four-part fracture in which the head is dislocated out of the glenoid and devoid of any soft-tissue attachments (high risk of osteonecrosis), this valgus impacted head will have a medial soft-tissue hinge with a lower risk of osteonecrosis. It is most amenable to open reduction and internal fixation with minimal soft-tissue stripping techniques. Bone grafting may be necessary on occasion. Nonsurgical management for displaced proximal humeral fractures generally results in a poor outcome. This patient does not have a humeral head defect. A hemiarthroplasty is not indicated.
REFERENCES: Jakob RP, Miniaci A, Anson PS, et al: Four-part valgus impacted fractures of the proximal humerus. J Bone Joint Surg Br 1991;73:295-298.
Resch H, Povacz P, Frohlich R, et al: Percutaneous fixation of three- and four-part fractures of the proximal humerus. J Bone Joint Surg Br 1997;79:295-300.
79. A 22-year-old professional baseball pitcher has had pain in the axillary region of his dominant shoulder for the past several weeks. While throwing a pitch during a game, he notes a sharp pulling sensation with a “pop” in his shoulder. Examination the following day reveals tenderness along the posterior axillary fold and pain and weakness with resisted extension of the shoulder. What is the most likely cause of his symptoms?
1- Type 2 tear of the superior labrum anterior and posterior
2- Tear of the anterior labrum
3- Tear of the subscapularis tendon
4- Tear of the latissimus dorsi tendon
5- Tear of the supraspinatus tendon
PREFERRED RESPONSE: 4
DISCUSSION: Injury to the latissimus dorsi tendon recently has been reported as a cause of pain in the thrower’s shoulder. The etiology of this injury is felt to be eccentric overload during the follow-through of the throwing motion. Recommended management for this unusual injury consists of a short period of rest, followed by physical therapy to restore shoulder motion and strength. Throwing is allowed when the athlete demonstrates full, pain-free motion and good strength and balance of the rotator cuff and scapular rotator muscles. Currently there are no defined indications for surgical repair.
REFERENCES: Schickendantz MS, Ho CP, Keppler L, et al: MR imaging of the thrower’s shoulder: Internal impingement, latissimus dorsi/subscapularis strains and related injuries.
Magn Reson Imaging Clin N Am 1999;7:39-49.
Livesey JP, Brownson P, Wallace WA: Traumatic latissimus dorsi: Tendon rupture. J Shoulder Elbow Surg 2002;11:642-644.
80. When comparing the addition of a trough at the greater tuberosity to direct repair of cortical bone, simulated rotator cuff repair in animal models has shown what type of change in the strength of the repair?
1- 0% increase
2- 25% increase
3- 50% increase
4- 25% decrease
5- 50% decrease
PREFERRED RESPONSE: 1
DISCUSSION: There was no difference observed in the healing of tendon to bone when comparing healing to cortical bone and to a cancellous trough.
REFERENCE: St Pierre P, Olson EJ, Elliott JJ, et al: Tendon healing to cortical bone compared with healing to a cancellous trough. J Bone Joint Surg Am 1995;77:1858-1866.
81. Figures 35a and 35b show the radiographs of a 20-year-old man who is unable to rotate his dominant forearm. Examination reveals that the arm is fixed in supination. To regain motion, management should consist of
1- observation.
2- dynamic splinting.
3- resection of the synostosis.
4- proximal radial excision.
5- forearm osteotomy.
PREFERRED RESPONSE: 3
DISCUSSION: The patient has a proximal synostosis; therefore, resection of the synostosis is considered the best option to regain motion. While forearm osteotomy can place the hand in a more functional position, rotation will not be restored. Proximal radial excision can provide forearm rotation; however, this procedure is reserved for patients who have a proximal radioulnar synostosis that is too extensive to allow a safe resection, involves the articular surface, and is associated with an anatomic deformity. Motion will not be restored with dynamic splinting.
REFERENCES: Kamineni S, Maritz NG, Morrey BF: Proximal radial resection for posttraumatic radioulnar synostosis: A new technique to improve forearm rotation. J Bone Joint Surg Am 2002;84:745-751.
Jupiter JB, Ring D: Operative treatment of post-traumatic proximal radioulnar synostosis.
J Bone Joint Surg Am 1998;80:248-257.
82. A 20-year-old-man sustained a scapular fracture after attempting to grab a beam as he fell through a ceiling at a job site 3 months ago. A clinical photograph is shown in Figure 36. He now reports pain in the anterior shoulder and difficulty with overhead activities. What nerve roots make up the involved peripheral nerve?
1- C3-T1
2- C4-C5
3- C5-C7
4- C6-C8
5- C8-T1
PREFERRED RESPONSE: 3
DISCUSSION: The patient sustained an injury to the long thoracic nerve, which supplies the serratus anterior. Branches of C5 and C6 enter the scalenus medius, unite in the muscle, and emerge as a single trunk and pass down the axilla. On the surface of the serratus anterior, the long thoracic nerve is joined by the branch from C7 and descends in front of the serratus anterior, providing segmental innervation to the serratus anterior.
REFERENCE: Leffert RD: Anatomy of the Brachial Plexus in Brachial Plexus Injuries. Churchill Livingstone, New York, NY, 1985.
83. A 20-year-old collegiate baseball pitcher has persistent deep shoulder pain. Examination reveals normal strength, 130 degrees of external rotation in abduction, 10 degrees of internal rotation in abduction, mild dynamic scapular winging, and equivocal findings on provocative tests for labral tears. Management should consist of
1- shoulder arthroscopy and repair of a possible labral tear.
2- anterior capsulorrhaphy for subtle anterior instability.
3- diagnostic arthroscopy of the glenohumeral joint and arthroscopic subacromial decompression.
4- rest from pitching, initiation of a rehabilitation program to restore internal rotation, and scapular stabilization.
5- rest from pitching and a corticosteroid injection into the subacromial space.
PREFERRED RESPONSE: 4
DISCUSSION: Although management of shoulder pain in the throwing athlete is controversial, there are some general principles. Initial management generally includes rest from throwing, restoring normal joint function, specifically motion and strength as well as eliminating pain. In this patient, examination reveals excessive external rotation and decreased internal rotation. This pattern is common in pitchers; however, the total arc of motion should remain close to 180 degrees in abduction. In this patient, the total arc is 140 degrees. Treatment should first focus on restoring a 180-degree arc with posterior scapular stretching, as well as pain control and muscle rehabilitation. Injections and surgery are generally reserved for patients who fail to respond to rest and rehabilitation.
REFERENCE: Burkhart SS, Morgan CD, Kibler WB: Shoulder injuries in overhead athletes: The “dead arm” revisited. Clin Sports Med 2000;19:125-158.
84. Which of the following best describes the most common anatomic variation seen in the glenoid labrum and the middle glenohumeral ligament in the anterosuperior quadrant of the shoulder?
1- Labrum attached to the glenoid rim and a flat/broad middle glenohumeral ligament
2- Sublabral hole with the labrum absent and a flat/broad middle glenohumeral ligament
3- Sublabral hole with a cord-like labrum and a flat/broad middle glenohumeral ligament
4- Anterosuperior labrum confluent with a cord-like middle glenohumeral ligament and no labral attachment to bone
5- Anterosuperior labrum confluent with a cord-like middle glenohumeral ligament and glenoid deficiency
PREFERRED RESPONSE: 1
DISCUSSION: Wide variations in the anatomy of the anterosuperior portion of the labrum and the middle glenohumeral ligament have been reported and are more common than previously thought. The labrum attached to the glenoid rim and a flat/broad middle glenohumeral ligament is the most common “normal” variation. A cord-like middle glenohumeral ligament is often associated with the presence of a sublabral hole. An anterosuperior labrum confluent with a cord-like middle glenohumeral ligament and no labral attachment to bone is the configuration of the Buford complex. The prevalence of each variation from one recent study is as follows: #1: 86.6%; #2: 3.3%; #3: 8.6%; and #4: 1.5%.
REFERENCES: Rao AG, Kim TK, Chronopoulos E, et al: Anatomical variants in the anterosuperior aspect of the glenoid labrum. J Bone Joint Surg Am 2003;85:653-659.
Ilahi OA, Labbe MR, Cosculluela P: Variants of the anterosuperior glenoid labrum and associated pathology. Arthroscopy 2002;18:882-886.
Williams MM, Snyder SJ, Buford D: The Buford complex-The “cord-like” middle glenohumeral ligament and absent anterosuprior labrum complex: A normal anatomic capsulolabral variant. Arthroscopy 1994;10:241-247.
85. A 21-year-old hockey player who has recurrent shoulder subluxations undergoes an anterior capsulorrhaphy under general anesthesia, and an interscalene block is used to relieve postoperative pain. At the 1-week follow-up examination, he reports loss of sensation over the lateral region of the shoulder and is unable to actively contract the deltoid muscle. The remainder of the examination is normal. What is the best course of action at this time?
1- Early exploration and possible repair of the axillary nerve
2- Urgent electromyography to assess for level of nerve injury
3- Continued normal postoperative care and observation of the nerve injury
4- Consultation with the anesthesiologist regarding a complication of the interscalene block
5- MRI to evaluate for a possible hematoma compressing the neurovascular bundle
PREFERRED RESPONSE: 3
DISCUSSION: The patient has an axillary nerve injury, which is relatively uncommon after surgery for instability. This type of injury generally is the result of a stretch injury rather than transection or a hematoma. Therefore, observation is indicated in the early postoperative period. After approximately 6 weeks, electromyography can be used to confirm and document the point of injury. Interscalene blocks can cause prolonged nerve injury but usually are not limited to the axillary nerve.
REFERENCE: Ho E, Cofield RH, Balm MR, et al: Neurologic complications of surgery for anterior shoulder instability. J Shoulder Elbow Surg 1999;8:266-270.
86. A 10-year-old boy has had a prominent scapula for the past year. He reports crepitus and aching over the area, but only when he is active. A radiograph and CT scans are shown in Figures 37a through 37c. What is the most likely diagnosis?
1- Enchondroma
2- Osteochondroma
3- Chondroblastoma
4- Ewing’s sarcoma
5- Chondrosarcoma
PREFERRED RESPONSE: 2
DISCUSSION: The findings are typical for an osteochondroma. It is found as an outgrowth of bone and cartilage from those bones that arise from enchondral ossification. It may be flat, verrucous, or with a long stalk and cauliflower-like cap. Osteochondromas can become symptomatic secondary to irritation of the adjacent musculature. They cease to proliferate when epiphyseal growth ceases.
REFERENCE: Schmade GA, Conrad EV III, Raskind WH: The natural history of hereditary multiple exostoses. J Bone Joint Surg Am 1994;76:986-992.
87. In patients who have undergone nonsurgical management for idiopathic adhesive capsulitis, long-term follow-up studies have shown which of the following results?
1- Continued night pain
2- Continuous decrease in forward elevation over time
3- Resolution of most symptoms, with range of motion equal to the contralateral shoulder
4- Decreased range of motion compared with the contralateral shoulder
5- Decreased pain compared with the contralateral shoulder
PREFERRED RESPONSE: 4
DISCUSSION: Results have been satisfactory in many patients; however, at long-term follow-up, examination of the affected shoulder often shows some decrease in range of motion compared with the contralateral side. Although range of motion often improves over time, it does not return to normal in 60% of patients. Pain improves but is often increased compared with the contralateral side.
REFERENCES: Griggs SM, Ahn A, Green A: Idiopathic adhesive capsulitis: A prospective functional outcome study of nonoperative treatment. J Bone Joint Surg Am 2000;82:1398-1407.
Shaffer B, Tibone JE, Kerlan RK: Frozen shoulder: A long-term follow-up. J Bone Joint Surg Am 1992;74:738-746.
88. Which of the following statements best describes the relationship between tissue response to thermal capsulorrhaphy and the type of device used?
1- Laser devices produce photon excitation, which produces less effect than electromagnetic devices.
2- Bipolar devices cause less tissue damage than monopolar devices.
3- Monopolar devices cause less tissue damage than bipolar devices.
4- The tissue response is the same regardless of the device.
5- Tissue response is greatest with bipolar devices when not in ablate mode.
PREFERRED RESPONSE: 4
DISCUSSION: Although radiofrequency devices and lasers differ fundamentally in the way they generate heat within a tissue, both classes of devices are capable of producing temperatures within the critical temperature range (65 to 75 degrees C) for collagen denaturation and subsequent tissue shrinkage. When it comes to cell viability and tissue response, heat is heat. Once critical temperatures are reached, cells will die at 45 degrees C, collagen will become denatured at 60 degrees C, and tissue ablation will occur at 100 degrees C no matter what the source of thermal energy. Therefore, claims of a better or different type of heat have little bearing on the biologic response of the tissue. Histologic, ultrastructural, and biomaterial alterations induced by laser and radiofrequency energy have been shown to be similar.
REFERENCES: Arnoczky SP, Aksan A: Thermal modification of connective tissues: Basic science considerations and clinical implications. J Am Acad Orthop Surg 2000;8:305-313.
Hayashi K, Markel MD: Thermal modification of joint capsule and ligamentous tissues: The use of thermal energy in sports medicine. Operative Techniques Sports Med 1998;6:120-125.
Naseef GS III, Foster TE, Trauner K, et al: The thermal properties of bovine joint capsule: The basic science of laser- and radiofrequency-induced capsular shrinkage. Am J Sports Med 1997;25:670-674.
89. A 35-year-old man has profound deltoid weakness after sustaining a traumatic anterior shoulder dislocation 6 weeks ago. Electromyographic (EMG) studies confirm an axillary nerve injury. Follow-up examination at 3 months reveals no recovery of function. What is the best course of action?
1- Surgical repair of the Bankart lesion
2- Exploration of the axillary nerve
3- MRI neurography
4- Repeat EMG studies
5- Continued observation and physical therapy
PREFERRED RESPONSE: 4
DISCUSSION: Documenting the status of recovery at this time is appropriate; therefore, repeat EMG studies should be conducted to check for early signs of reinnervation. Timing of nerve exploration in this setting is debated, with authors suggesting exploration if there is no sign of recovery at 6 to 9 months.
REFERENCES: Perlmutter GS: Axillary nerve injury. Clin Orthop 1999;368:28-36.
Artico M, Salvati M, D’Andrea V, et al: Isolated lesions of the axillary nerves: Surgical treatment and outcome in twelve cases. Neurosurgery 1991;29:697-700.
Vissar CP, Coene LN, Brand R, et al: The incidence of nerve injury in anterior dislocation of the shoulder and its influence on functional recovery: A prospective clinical and EMG study. J Bone Joint Surg Br 1999;81:679-685.
Pasila M, Jarma H, Kiviluoto O, et al: Early complications of primary shoulder dislocations. Acta Orthop Scand 1978;49:260-263.
90. A 65-year-old woman has had chronic aching discomfort involving her elbow for the past 6 months. Radiographs and a biopsy specimen are shown in Figures 38a through 38c. What is the most likely diagnosis?
1- Metastatic carcinoma (thyroid)
2- Multiple myeloma
3- Lymphoma
4- Synovial cell sarcoma
5- Osteomyelitis
PREFERRED RESPONSE: 2
DISCUSSION: The histologic features of multiple myeloma are distinctive for this lesion. The plasma cells are round or oval and have an eccentric nucleus and prominent nucleolus. These characteristics and a clear area next to the eccentric nucleus representing the prominent Golgi center are pathognomonic for plasma cells. Lymphoma is in the differential diagnosis; the most frequent types that occur in bone are large cell or mixed small and large cell types. The histologic appearance of the specimen is not consistent with the other choices.
REFERENCE: Dorfman HD, Bodgan C: Immunohematopoietic tumors, in Dorfman HD, Bogdan C (eds): Bone Tumors. St Louis, MO, Mosby, 1998, Chapter 12.
91. Which of the following clinical findings is commonly associated with symptomatic partial-thickness rotator cuff tears?
1- Negative impingement signs
2- Abnormal lift-off test
3- External rotation lag sign
4- Painful arc with active range of motion
5- Mismatch in active and passive motion
PREFERRED RESPONSE: 4
DISCUSSION: In symptomatic partial-thickness rotator cuff tears, a painful arc with active range of motion is common, impingement signs are usually positive, and the lift-off test is normal. Active and passive range of motion measurements are often equal, although active range of motion can be painful. External rotation lag signs are often seen with larger full-thickness tears.
REFERENCES: Hertel R, Ballmer FT, Lambert SM, Gerber C: Lag signs in the diagnosis of rotator cuff rupture. J Shoulder Elbow Surg 1996;5:307-313.
McConville OR, Iannotti JP: Partial thickness tears of the rotator cuff: Evaluation and management. J Am Acad Orthop Surg 1999;7:32-43.
Gerber C, Krushell RJ: Isolated rupture of the tendon of the subscapularis muscle: Clinical features in 16 cases. J Bone Joint Surg Br 1991;73:389-394.
Fukuda H: Partial-thickness rotator cuff tears: A modern view on Codman’s classic. J Shoulder Elbow Surg 2000;9:163-168.
92. A 65-year-old woman landed on her nondominant left shoulder in a fall. An AP radiograph is shown in Figure 39. Management should consist of
1- closed reduction and immobilization.
2- closed reduction and percutaneous pinning.
3- open reduction and internal fixation.
4- humeral hemiarthroplasty with tuberosity repair.
5- total shoulder arthroplasty.
PREFERRED RESPONSE: 4
DISCUSSION: The radiograph reveals a four-part fracture-dislocation of the proximal humerus. Humeral hemiarthroplasty and tuberosity repair is the treatment of choice because the risk of osteonecrosis is high after attempted repair of this injury. Glenoid resurfacing is reserved for acute fractures in which there is significant preexisting glenoid arthrosis, such as in patients with rheumatoid arthritis.
REFERENCES: Neer CS II: Displaced proximal humeral fractures: II. Treatment of three- and four-part displacement. J Bone Joint Surg Am 1970;52:1090-1103.
Robinson CM, Page RS, Hill RM, et al: Primary hemiarthroplasty for treatment of proximal humeral fractures. J Bone Joint Surg Am 2003;85:1215-1223.
93. To avoid damage to the ascending branch of the anterior humeral circumflex artery during open reduction and internal fixation of a proximal humeral fracture, the blade plate should be placed in what position?
1- Medial to the bicipital groove and pectoralis major tendon
2- Medial to the bicipital groove and lateral to the pectoralis major tendon
3- Lateral to the bicipital groove and pectoralis major tendon
4- Lateral to the bicipital groove and medial to the pectoralis major tendon
5- In the bicipital groove
PREFERRED RESPONSE: 3
DISCUSSION: The pectoralis major tendon inserts lateral to the biceps tendon, which runs in the bicipital groove. The primary vascular supply of the articular surface of the humeral head is derived from the anterior circumflex humeral artery, which continues into the arcuate artery once it enters the bone. The entry point is on the anterolateral aspect of the humerus just medial to the greater tuberosity within the bicipital groove. To avoid compromising circulation, the blade plate should be placed lateral to the bicipital groove and pectoralis major tendon insertion.
REFERENCES: Loebenberg M, Plate AM, Zuckerman J: Osteonecrosis of the humeral head. Instr Course Lect 1999;48:349-357.
Gerber C, Schneeberger AG: The arterial vascularization of the humeral head: An anatomical study. J Bone Joint Surg Am 1990;72:1486-1494.
94. An otherwise healthy 13-year-old boy sustains the fracture shown in Figure 40 while throwing a fastball. Management should consist of
1- an arm sling.
2- functional bracing supporting the humerus and arm.
3- closed reduction and a shoulder spica cast.
4- open reduction and internal fixation with retrograde rods.
5- open reduction and internal fixation with a rigid plate and screws.
PREFERRED RESPONSE: 2
DISCUSSION: Nonsurgical management such as a functional brace, hanging arm cast, or sugar tong splint is the treatment of choice for a fracture of the humeral shaft that is the result of throwing. The fracture surface typically is wide and the degree of displacement is not large; therefore, surgery is not indicated in most patients.
REFERENCES: Ogawa K, Yoshida A: Throwing fracture of the humeral shaft: An analysis of 90 patients. Am J Sports Med 1998;26:242-246.
Kaplan H, Kiral A, Kuskucu M, et al: Report of eight cases of humeral fracture following the throwing of hand grenades. Arch Orthop Trauma Surg 1998;117:50-52.
95. A 24-year-old man sustains an injury to his right elbow after falling 10 feet. Radiographs are shown in Figures 41a and 41b. Treatment should consist of
1- open reduction and internal fixation, followed by casting.
2- open reduction and internal fixation, followed by early range of motion.
3- open reduction and internal fixation, medial collateral ligament repair, and early range of motion.
4- open reduction and internal fixation of the ulna, application of a hinged external fixator, and early range of motion.
5- closed reduction and splinting, followed by early range of motion.
PREFERRED RESPONSE: 2
DISCUSSION: Transolecranon fracture-dislocations are most effectively managed with open reduction and internal fixation, followed by early aggressive range of motion. Concomitant injury to the collateral ligament is rare, and stability is achieved by anatomic reconstruction of the olecranon fracture with rigid fixation. The need for collateral ligament repair or a hinged external fixator is uncommon in this fracture pattern.
REFERENCE: Ring D, Jupiter JB, Sanders RW, et al: Transolecranon fracture-dislocation of the elbow. J Orthop Trauma 1997;11:545-550.
96. After closed reduction of the dislocation shown in Figure 42, it is essential to avoid placing the upper extremity in what position for the first 4 to 6 weeks?
1- Abduction
2- External rotation
3- Internal rotation
4- Extension
5- Elevation
PREFERRED RESPONSE: 3
DISCUSSION: Acute posterior dislocations occur rarely, accounting for less than 5% of acute dislocations. They are most often the result of falls on an outstretched hand. Reduction can be accomplished with flexion of the arm to 90 degrees and adduction to disimpact the humeral head from the glenoid rim. The arm is then externally rotated until the head has cleared the glenoid rim. Following brace immobilization in neutral to 5 to 10 degrees of external rotation and slight abduction, it is critical to avoid internal rotation for 4 to 6 weeks.
REFERENCES: Burkhead WZ Jr, Rockwood CA Jr: Treatment of instability of the shoulder with an exercise program. J Bone Joint Surg Am 1986;68:724-731.
Pollock RG, Bigliani LU: Recurrent posterior shoulder instability: Diagnosis and treatment.
Clin Orthop 1993;291:85-96.
97. A baseball player has had diffuse scapular soreness for the past 8 weeks. He reports that it began insidiously over several days and gradually has become worse. He denies any history of trauma. Examination reveals drooping of the shoulder, with lateral winging of the scapula at rest. He is otherwise neurologically intact. What is the best course
of action?
1- Immediate MRI of the brain
2- Electromyography and nerve conduction velocity studies
3- Physical therapy and observation
4- Lyme titer
5- Psychiatric consultation
PREFERRED RESPONSE: 2
DISCUSSION: Lateral scapular winging is characteristic of trapezius palsy, whereas medial scapular winging is characteristic of long thoracic nerve palsy. During sports activity, injury to the spinal accessory nerve is rare but may occur with blunt or stretching trauma. Patients often report an asymmetric neckline, drooping shoulder, winging of the scapula, and weakness of forward elevation. Evaluation should include a complete electrodiagnostic examination.
REFERENCES: Wiater JM, Bigliani LU: Spinal accessory nerve injury. Clin Orthop 1999;368:5-16.
Wiater JM, Flatow EL: Long thoracic nerve injury. Clin Orthop 1999;368:17-27.
Mariani PP, Santoriello P, Maresca G: Spontaneous accessory nerve palsy. J Shoulder Elbow Surg 1998;7:545-546.
Porter P, Fernandez GN: Stretch-induced spinal accessory nerve palsy: A case report. J Shoulder Elbow Surg 2001;10:92-94.
Cohn BT, Brahms MA, Cohn M: Injury to the eleventh cranial nerve in a high school wrestler. Orthop Rev 1986;15:59-64.
98. Which of the following best describes the mechanical response of the inferior glenohumeral ligament to repetitive subfailure strains?
1- Decreased peak load response and length decreases
2- Decreased peak load response and recoverable length increases
3- Decreased peak load response and unrecoverable length increases
4- Increased peak load response and recoverable length increases
5- Increased peak load response and unrecoverable length increases
PREFERRED RESPONSE: 3
DISCUSSION: Repetitive subfailure strains have been shown to affect the mechanical behavior of the inferior glenohumeral ligament, producing dramatic declines in the peak load response and length increases that are largely unrecoverable. In another study, anteroinferior subluxation was found to result in nonrecoverable strain in the anteroinferior capsule, varying from 3% to 7% through a range of joint subluxation.
REFERENCES: Pollock RG, Wang VM, Bucchieri JS, et al: Effects of repetitive subfailure strains on the mechanical behavior of the inferior glenohumeral ligament. J Shoulder Elbow Surg 2000;9:427-435.
Malicky DM, Kuhn JE, Frisancho JC, et al: Nonrecoverable strain fields of the anteroinferior glenohumeral capsule under subluxation. J Shoulder Elbow Surg 2002;11:529-540.
99. A 38-year-old woman who tripped and fell on her outstretched arm reports pain with movement. Examination reveals swelling. AP and lateral radiographs are shown in Figures 43a and 43b. Management should consist of
1- excision of the fracture fragment.
2- radial head replacement.
3- closed reduction and cast immobilization.
4- open reduction and internal fixation of the radial head.
5- open reduction and internal fixation of the capitellum.
PREFERRED RESPONSE: 5
DISCUSSION: The patient has a type I (Hahn-Steinthal) capitellar fracture that is best seen on the lateral radiograph. If a fracture fragment is seen proximal to the radial head, a capitellar fracture is the most likely injury because radial head fractures do not migrate proximally. The fragment is large enough for fixation. Excision is the preferred treatment for small shear osteochondral type II (Kocher-Lorenz) capitellar fractures. Closed reduction usually is not successful because of rotation of the displaced fragment.
REFERENCES: Mehdian H, McKee M: Management of proximal and distal humerus fractures. Orthop Clin North Am 2000;31:115-127.
Ring D, Jupiter J, Gulotta L: Articular fractures of the distal part of the humerus. J Bone Joint Surg Am 2003;85:232-238.
100. A 15-year-old girl reports popping and clicking at the sternoclavicular joint and an intermittent asymmetrical prominence of the medial head of the clavicle. She denies any history of trauma or other symptoms. Management should consist of
1- figure-of-8 splinting to maintain the clavicle in a reduced position.
2- an exercise program to stabilize the joint in a reduced position.
3- elective reconstruction of the sternoclavicular joint ligaments.
4- reassurance and local symptomatic treatment.
5- closed reduction and temporary pin fixation to stabilize the joint.
PREFERRED RESPONSE: 4
DISCUSSION: Atraumatic subluxation or dislocation of the sternoclavicular joint typically occurs in individuals with generalized ligamentous laxity. It is generally not painful, has no long-term sequelae, and needs no treatment. In fact, it is more likely to be painful following surgery than if managed nonsurgically.
REFERENCES: Rockwood CA Jr, Odor JM: Spontaneous atraumatic anterior subluxation of the sternoclavicular joint. J Bone Joint Surg Am 1989;71:1280-1288.
Rockwood CA, Matsen FA (eds): The Shoulder, ed 2. Philadelphia, PA, WB Saunders, 1998,
p 583.
101. A 45-year-old woman who recently underwent biopsy of a lymph node in the right posterior cervical triangle now finds it difficult to hold objects overhead and has diffuse aching in the right shoulder region. What is the most likely diagnosis?
1- Rotator cuff tear
2- Rhomboid paralysis
3- Deltoid paralysis
4- Triceps paralysis
5- Trapezius paralysis
PREFERRED RESPONSE: 5
DISCUSSION: The trapezius is innervated by the spinal accessory nerve. The nerve is superficial in the area of the posterior cervical triangle and is prone to injury during dissection. Paralysis of the trapezius causes loss of scapular stability when forward flexion or abduction of the shoulder is attempted.
REFERENCES: Vastamaki M, Solonen KA: Accessory nerve injury. Acta Orthop Scand 1984;55:296-299.
Leffert RD: Neurological Problems, in Rockwood CA, Matsen FA (eds): The Shoulder. Philadelphia, PA, WB Saunders, 1990, vol 2, pp 759-760.
102. The posterior cord of the brachial plexus terminates into what two main branches?
1- Lateral and medial pectorals
2- Lateral pectoral and suprascapular
3- Radial and axillary nerves
4- Musculocutaneous and the lateral root of the median nerve
5- Ulnar and medial roots of the median nerve
PREFERRED RESPONSE: 3
DISCUSSION: The posterior cord of the brachial plexus terminates into the radial and axillary nerves. The lateral cord terminates in branches to the musculocutaneous and the lateral root of the median nerve. The medial cord terminates in branches to the ulnar and medial roots of the median nerve.
REFERENCE: Hollinshead WH: Anatomy for Surgeons: The Back and Limbs, ed 3. Philadelphia, PA, Harper and Row, 1982, pp 228-236.
103. Atraumatic neuropathy of the suprascapular nerve usually occurs at what
anatomic location?
1- Suprascapular and spinoglenoid notches
2- Omohyoid muscle
3- Anterior trapezius muscle
4- Infraspinatus fascia
5- Teres minor superior border
PREFERRED RESPONSE: 1
DISCUSSION: The suprascapular nerve passes through the suprascapular notch and the spinoglenoid notch before innervating the infraspinatus muscle. At both locations, the suprascapular nerve is prone to nerve compression, which often results from a ganglion cyst. The other anatomic locations are not associated with suprascapular nerve impingement.
REFERENCES: Romeo AA, Rotenberg DD, Bach BR: Suprascapular neuropathy. J Am Acad Orthop Surg 1999;7:358-367.
Post M, Mayer J: Suprascapular nerve entrapment: Diagnosis and treatment. Clin Orthop 1987;223:126-136.
104. A 22-year-old patient underwent successful reduction of a posterolateral elbow dislocation. Management should now consist of
1- splinting for 5 weeks.
2- active range-of-motion exercises after 1 to 3 days.
3- delayed passive stretching at 2 weeks.
4- open medial collateral ligament reconstruction.
5- open lateral collateral ligament reconstruction.
PREFERRED RESPONSE: 2
DISCUSSION: The elbow usually is stable after reduction in most elbow dislocations. Ross and associates reported that supervised motion begun immediately after reduction was effective in uncomplicated dislocations. The elbow will become stiff if immobilization is applied for an extended period of time. Immediate open treatment is not indicated for a simple elbow dislocation.
REFERENCES: Ross G, McDevitt ER, Chronister R, et al: Treatment of simple elbow dislocation using an immediate motion protocol. Am J Sports Med 1999;27:308-311.
O’Driscoll SW, Jupiter JB, King GJ, et al: The unstable elbow. J Bone Joint Surg Am 2000;82:724-738.
105. A 56-year-old woman who underwent axillary node dissection 4 months ago now reports shoulder pain, weakness of forward elevation, and obvious winging of the scapula. What structure has been injured?
1- Long thoracic nerve
2- Spinal accessory nerve
3- Thoracodorsal nerve
4- Lower trunk of the brachial plexus
5- Posterior cord of the brachial plexus
PREFERRED RESPONSE: 1
DISCUSSION: The long thoracic nerve, which innervates the serratus anterior, is prone to injury because of its superficial location along the chest wall. The long thoracic nerve is derived from the roots of C5, C6, and C7. The spinal accessory nerve innervates the trapezius, and the thoracodorsal nerve innervates the latissimus dorsi. The posterior cord of the brachial plexus provides the axillary and the radial nerves.
REFERENCES: Hollinshead WH: Anatomy for Surgeons: The Back and Limbs, ed 3. Philadelphia, PA, Harper and Row, 1982, pp 259-340.
Marmor L, Bechtal CO: Paralysis of the serratus anterior due to electric shock relieved by transplantation of the pectoralis major muscle: A case report. J Bone Joint Surg Am 1983;45:156-160.
106. The lateral arm flap is based on what arterial supply?
1- Posterior radial collateral
2- Anterior radial collateral
3- Brachial
4- Subscapular
5- Circumflex scapular
PREFERRED RESPONSE: 1
DISCUSSION: The lateral arm flap is based on the posterior radial collateral artery, a branch of the profunda brachial artery.
REFERENCES: Katsaros J, Tan E, Zoltie N: The use of the lateral arm flap in upper limb surgery. J Hand Surg 1991;16:598-604.
Katsaros J, Schusterman M, Beppu M, et al: The lateral upper arm flap: Anatomy and clinical applications. Ann Plast Surg 1984;12:489-499.
107. A 32-year-old man has a closed oblique displaced fracture at the junction of the lower and middle third of the humeral shaft and a complete radial nerve palsy. Closed reduction is performed and is felt to be acceptable. Management of the radial nerve palsy should consist of
1- exploration and repair of the radial nerve if clinical findings or electromyographic studies show no improvement at 2 to 3 weeks.
2- exploration and repair of the radial nerve if clinical findings or electromyographic studies show no improvement at 14 weeks.
3- transfer of the pronator teres to the extensor carpi radialis brevis if clinical findings or electromyographic studies show no improvement at 14 weeks.
4- immediate exploration and repair of the radial nerve, along with internal fixation with a plate and screws.
5- immediate exploration and repair of the radial nerve, along with internal fixation with an intramedullary nail.
PREFERRED RESPONSE: 2
DISCUSSION: In patients who have radial nerve dysfunction associated with a closed humeral fracture, nerve function usually will return to normal without surgical exploration. If clinical findings or electromyographic studies show no improvement at 3 months, surgical exploration and repair can be performed. Tendon transfers are performed if nerve repair is deemed unsuccessful.
REFERENCES: Pollock FH, Drake D, Bovill EG, et al: Treatment of radial neuropathy associated with fractures of the humerus. J Bone Joint Surg Am 1981;63:239-243.
Norris TR (ed): Orthopaedic Knowledge Update: Shoulder and Elbow 2. Rosemont, IL, American Academy of Orthopaedic Surgeons, 2002, pp 237-247.
108. A 19-year-old man sustains a low-velocity gunshot wound to the forearm. What
factor most strongly correlates with the development of compartment syndrome
after this injury?
1- Fracture comminution
2- Fracture of both the radius and ulna
3- Fracture of the proximal third of the forearm
4- Fracture displacement of more than 10 mm
5- Retained bullet fragments
PREFERRED RESPONSE: 3
DISCUSSION: In a multivariate analysis, the strongest factor for the development of compartment syndrome is fracture of the proximal third of the forearm. However, compartment syndrome can still occur without a fracture. Therefore, these patients should be followed with a high level of suspicion for the development of compartment syndrome.
REFERENCES: Moed BR, Fakhouri AJ: Compartment syndrome after low-velocity gunshot wounds to the forearm. J Orthop Trauma 1991;5:134-137.
Hahn M, Strauss E, Yang EC: Gunshot wounds to the forearm. Orthop Clin North Am 1995;26:85-93.
109. A 30-year-old farmer undergoes replantation of an above-the-elbow amputation. What form of management is most important following this surgery?
1- High volume diuresis with alkalinization of the urine
2- Systemic heparinization of the patient for 72 hours
3- Elevation of the extremity, with maintanence of the patient’s room temperature at 80 degrees F (26.6 degrees C)
4- Satellite ganglion continuous sympathetic block
5- Daily IV administration of low-molecular-weight dextran
PREFERRED RESPONSE: 1
DISCUSSION: After major limb replantation, the occurrence of ischemic rhabdomyonecrosis can result in lactic acidosis and myoglobulinemia. These complications can be limited by rapid repair of the arterial supply, potentially using a shunt before skeletal stability. Repair of the venous system should be performed after repair of the artery. High volume fluid replacement will maintain a diuresis, thus limiting the complications from myoglobulinemia.
REFERENCES: Wood MB: Replantations about the elbow, in Morrey BF (ed): The Elbow and Its Disorders. Philadelphia, PA, WB Saunders, 1985, pp 472-480.
Goldner RD, Nunley JA: Replantation proximal to the wrist, in Wood MD (ed) Hand Clinics: Microsurgery. Philadelphia, PA, WB Saunders, 1992, pp 413-425.
110. Figures 44a through 44c show the radiographs of an 18-year-old female soccer player who fell on her outstretched hand 1 day ago. She denies any history of wrist pain. Examination reveals tenderness at the anatomic snuffbox. Management should consist of
1- a long arm cast for 6 weeks, followed by a short arm cast for 6 weeks.
2- vascularized bone graft from the 1,2 intercompartmental supraretinacular artery.
3- open reduction and internal fixation with a differential pitch screw via a dorsal approach.
4- open reduction and internal fixation with a differential pitch screw via a volar approach
5- a removable thumb spica splint.
PREFERRED RESPONSE: 3
DISCUSSION: The treatment of choice for proximal pole scaphoid fractures is open reduction and internal fixation with a differential pitch screw via a dorsal approach. Healing rates of 100% have been reported for these acute fractures. Casting results in slow healing, with recommendations including 16 weeks or more in a cast. Vascularized bone grafts are not indicated for acute fractures.
REFERENCES: Rettig ME, Raskin KB: Retrograde compression screw fixation of acute proximal pole scaphoid fractures. J Hand Surg 1999;24:1206-1210.
Raskin KB, Parisi D, Baker J, et al: Dorsal open repair of proximal pole scaphoid fractures. Hand Clin 2001;17:601-610.
111. An excessively large radial styloidectomy poses a risk for wrist instability. What ligament is at greatest risk for injury?
1- Long radiolunate
2- Short radiolunate
3- Radioscaphocapitate
4- Scapholunate
5- Dorsal radiocarpal
PREFERRED RESPONSE: 3
DISCUSSION: The radioscaphocapitate ligament is the most radial of the extrinsic volar ligaments of the wrist. It has a mean attachment to the radius 4 mm from the tip of the
radial styloid.
REFERENCES: Nakamura T, Cooney WP III, Lui WH, et al: Radial styloidectomy: A biomechanical study on the stability of the wrist joint. J Hand Surg Am 2001;26:85-93.
Siegel DB, Gelberman RH: Radial styloidectomy: An anatomical study with special reference to radiocarpal intracapsular ligamentous morphology. J Hand Surg Am 1991;16:40-44.
112. What joint always remains uninvolved in all stages of scapholunate advanced collapse (SLAC) deformity of the wrist?
1- Distal radioscaphoid
2- Proximal radioscaphoid
3- Radiolunate
4- Scaphocapitate
5- Capitolunate
PREFERRED RESPONSE: 3
DISCUSSION: The development of arthritis in SLAC wrist follows a consistent pattern. Beginning at the radial styloid to the scaphoid articulation, it progresses through the entire radioscaphoid joint and the midcarpal joint. In all stages, the radiolunate joint is spared, which is the basis for a scaphoid excision and four-corner fusion performed as a motion-sparing procedure for treatment of this condition.
REFERENCES: Wyrick JD: Proximal row carpectomy and intercarpal arthrodesis for the management of arthritis. J Am Acad Orthop Surg 2003;11:277-281.
Watson HK, Ballett FL: The SLAC wrist: Scapholunate advanced collapse pattern of degenerative arthritis. J Hand Surg Am 1984;9:358-365.
Watson HK, Ryu J: Evolution of arthritis of the wrist. Clin Orthop 1986;202:57-67.
113. Free flap coverage for severe trauma to the upper extremity has the fewest complications when performed within what time period after injury?
1- 72 hours
2- 7 to 10 days
3- 2 weeks
4- 4 weeks
5- 4 months
PREFERRED RESPONSE: 1
DISCUSSION: Flap necrosis and infection rates are lowest if free flap coverage is performed within 72 hours of injury. Delays beyond 72 hours are associated with a higher rate of complications.
REFERENCES: Godina M: Early microsurgical reconstruction of complex trauma of the extremities. Plast Reconstr Surg 1986;78:285-292.
Manske PR (ed): Hand Surgery Update. Rosemont, IL, American Academy of Orthopaedic Surgeons, 1994, pp 31-37.
114. A 54-year-old woman with idiopathic carpal tunnel syndrome undergoes open carpal tunnel release with a flexor tenosynovectomy. The pathology from the tenosynovium is likely to show
1- fibrosis and edema.
2- polymorphonuclear cells.
3- negatively birefringent crystals.
4- macrophages and lymphocytes.
5- fibrinous degeneration of collagen fibers.
PREFERRED RESPONSE: 1
DISCUSSION: The tenosynovium excised at the time of a carpal tunnel release for idiopathic carpal tunnel syndrome rarely shows signs of acute or chronic inflammation. Fibrosis, edema, and vascular sclerosis are the most common histologic findings. A tenosynovectomy with a carpal tunnel release usually is not necessary in the treatment of idiopathic carpal tunnel syndrome.
REFERENCES: Shum C, Parisien M, Strauch RJ, et al: The role of flexor tenosynovectomy in the operative treatment of carpal tunnel syndrome. J Bone Joint Surg Am 2002;84:221-225.
Fuchs PC, Nathan PA, Myers LD: Synovial histology in carpal tunnel syndrome. J Hand Surg Am 1991;16:753-758.
Kerr CD, Sybert DR, Albarracin NS: An analysis of the flexor synovium in idiopathic carpal tunnel syndrome: Report of 625 cases. J Hand Surg Am 1992;17:1028-1030.
115. Examination of a 10-year-old girl with a hypoplastic breast and atrophic pectoralis major may also reveal which of the following findings?
1- Absent middle phalanx
2- Absent deltoid muscle
3- Absent radius
4- Contralateral hypoplastic thumb
5- Clubfoot deformity
PREFERRED RESPONSE: 1
DISCUSSION: Poland’s syndrome has four main features: 1) short digits as the result of absence or shortening of the middle phalanx; 2) syndactyly of the short digits usually consisting of a simple, complete type; 3) hypoplasia of the hand and forearm; and 4) absence of the sternocostal head of the pectoralis major on the same side.
REFERENCES: Wilson MR, Louis DS, Stevenson TR: Poland’s syndrome: Variable expression and associated anomalies. J Hand Surg 1988;13:880-882.
Poland A: Deficiency of the pectoralis muscle. Guys Hosp Rep 1841;6:191.
116. Figures 45a and 45b show the radiographs of a 40-year-old woman with rheumatoid arthritis who is unable to straighten her ring and little fingers. Examination reveals that the fingers can be passively corrected, but she is unable to actively maintain the fingers in extension. Management should consist of
1- radial head resection.
2- dynamic splinting.
3- metacarpophalangeal arthroplasties.
4- total wrist arthrodesis with a flexor digitorum sublimis to extensor digitorum communis transfer.
5- distal ulnar resection with an extensor indius proprius to extensor digitorum communis transfer.
PREFERRED RESPONSE: 5
DISCUSSION: The patient has extensor tendon ruptures at the level of the wrist that are the result of synovitis at the distal radioulnar joint (Vaughn-Jackson syndrome). Extensor indius proprius transfer appropriately matches strength and excursion of the ruptured extensor digiti quinti and extensor digitorum communis tendons. An extensor tenosynovectomy with distal radioulnar joint resection decreases the synovitis, which if left untreated may cause additional tendon ruptures. Radial head resection is used for posterior interosseous nerve compression secondary to radial head synovitis, and in this patient only two fingers are involved, which rules out this diagnosis. Dynamic splinting is not indicated for ruptured tendons. Metacarpophalangeal arthroplasties and imbrication of the sagittal bands are used for metacarpophalangeal arthritis and extensor tendon subluxation. If this was the problem, the patient should be able to maintain the fingers in extension after they are passively extended. Total wrist arthrodesis prevents the tenodesis effect, thus limiting effective tendon excursion and making the proposed transfer less effective.
REFERENCES: Feldon P, Terrono AL, Nalebuff EA, et al: Rheumatoid arthritis and other connective tissue diseases: Tendon ruptures, in Green DP, Hotchkiss RN, Pederson WC (eds): Green’s Operative Hand Surgery, ed 4. New York, NY, Churchill Livingstone, 1999,
pp 1669-1684.
Moore JR, Weiland AJ, Valdata L: Tendon ruptures in the rheumatoid hand: Analysis of treatment and functional results in 60 patients. J Hand Surg Am 1987;12:9-14.
Leslie BM: Rheumatoid extensor tendon ruptures. Hand Clin 1989;5:191-202.
117. Figures 46a through 46e show the radiographs of a 22-year-old man who injured his wrist in a motorcycle accident. He has no other injuries. What is the best course of action?
1- Radiolunate fusion
2- Open repair of the volar extrinsic wrist ligaments
3- Open reduction and internal fixation
4- Thumb spica cast immobilization for 6 weeks
5- External fixator
PREFERRED RESPONSE: 3
DISCUSSION: The patient has a fracture-dislocation of the radiocarpal joint. Attached to the large radial styloid fragment are the extrinsic wrist ligaments to the carpus. This injury should be treated with open reduction and internal fixation of the styloid fracture. Radiolunate fusion or extrinsic ligament repair is suggested when the extrinsic ligaments are ruptured, resulting in ulnar translocation of the carpus.
REFERENCES: Dumontier C, Meyer ZU, Reckendorf G, et al: Radiocarpal dislocations: Classification and proposal for treatment: A review of twenty-seven cases. J Bone Joint Surg Am 2001;83:212.
Bilos ZJ, Pankovich AM, Yelda S: Fracture-dislocation of the radiocarpal joint: A clinical study of five cases. J Bone Joint Surg Am 1977;59:198-203.
118. A 36-year-old nurse has had redness, pain, and small vesicles on the pulp of her middle finger for the past 3 days. Management should consist of
1- observation.
2- marsupialization of the nail fold.
3- application of copper sulfate.
4- application of calcium gluconate.
5- incision and drainage of the pulp.
PREFERRED RESPONSE: 1
DISCUSSION: Small vesicles on the fingers of a health care worker suggest a herpetic infection, and the management of choice is observation. Incision and drainage may result in a bacterial infection. Marsupialization is used in the treatment of a chronic paronychia. Calcium gluconate is used for hydrofluoric acid burns, and copper sulfate is used for white phosphorus burns.
REFERENCES: Fowler JR: Viral Infections. Hand Clin 1989;5:613-627.
Jebsen PL: Infections of the fingertip: Paronychias and felons. Hand Clin 1998;14:547-555.
119. A 35-year-old man has numbness and tingling in the index, middle, and ring fingers. History reveals that he also has had vague wrist pain and stiffness since being injured in a motorcycle accident 1 year ago. Radiographs are shown in Figures 47a through 47c. Management should consist of
1- splinting and injections for carpal tunnel syndrome.
2- scaphoid excision and four-bone fusion.
3- proximal row carpectomy via dorsal and volar incisions.
4- MRI of the wrist.
5- carpal tunnel release.
PREFERRED RESPONSE: 3
DISCUSSION: The patient has a chronic unrecognized volar lunate dislocation. Median nerve compression is the result of the lunate displaced into the carpal tunnel. The diagnosis can be made by radiographs; MRI is not necessary. A volar approach allows median nerve decompression with excision of the lunate, whereas a dorsal approach facilitates excision of the scaphoid and triquetrum.
REFERENCES: Rettig ME, Raskin KB: Long-term assessment of proximal row carpectomy for chronic perilunate dislocations. J Hand Surg Am 1999;24:1231-1236.
Howard FM, Dell PC: The unreduced carpal dislocation: A method of treatment. Clin Orthop 1986;202:112-116.
120. A 42-year-old woman has persistent thumb pain that she notes is worse with opening jars and turning her car key. Opponens splinting provides some relief, but she is poorly tolerant of the splint. Finkelstein’s test is negative, and a carpometacarpal grind test is positive. The radiographs shown in Figures 48a and 48b reveal minimal degenerative changes at the first carpometacarpal joint. What is the best course of action?
1- Arthroscopic debridement of the first carpometacarpal joint with thermal shrinkage of the volar capsule
2- Extension osteotomy of the first metacarpal
3- Arthrodesis of the first carpometacarpal joint
4- Denervation of the first carpometacarpal joint
5- Nerve conduction velocity studies
PREFERRED RESPONSE: 2
DISCUSSION: The woman has early basilar thumb arthritis. An extension osteotomy will redirect the force to the dorsal, more uninvolved portion of the first carpometacarpal joint and has been reported to alleviate pain in these patients. Arthrodesis is usually reserved for young, typically male laborers. Thermal shrinkage and denervation are considered experimental at this time. Interposition arthroplasty is typically used for more advanced stages of arthritis.
REFERENCES: Tomaino MM: Treatment of Eaton stage I trapeziometacarpal disease with thumb metacarpal extension osteotomy. J Hand Surg Am 2000;25:1100-1106.
Pellegrini VD Jr, Parentis M, Judkins A, et al: Extension metacarpal osteotomy in the treatment of trapeziometacarpal osteoarthritis: A biomechanical study. J Hand Surg Am 1996;21:16-23.
Tomaino MM, Pellegrini VD, Burton RI: Arthroplasty of the basal joint of the thumb: Long-term follow-up after ligament reconstruction with tendon interposition. J Bone Joint Surg Am 1995;77:346-355.
121. A 45-year-old man sustains a low-velocity gunshot wound to the base of the right thumb. The open wound is allowed to heal by secondary intention, resulting in a contracture of the first web space. Clinical photographs are shown in Figures 49a through 49c. Treatment should now consist of
1- Z-plasty.
2- a posterior interosseous fasciocutaneous flap.
3- a reverse cross-finger flap from the index finger.
4- excision of the contracture with placement of a full-thickness skin graft.
5- excision of the contracture with placement of a split-thickness skin graft.
PREFERRED RESPONSE: 2
DISCUSSION: The contracture is too large for a Z-plasty, which allows a 75% increase in length. Excision of the scar with placement of a skin graft is prone to contracture. A posterior interosseous fasciocutaneous flap will provide enough well-vascularized tissue and is well suited to reach the first dorsal web space.
REFERENCES: Buchler U, Frey HP: Retrograde posterior interosseous flap. J Hand Surg Am 1991;16:283-292.
Brunelli F, Valenti P, Dumontier C, et al: The posterior interosseous reverse flap: Experience with 113 flaps. Ann Plast Surg 2001;47:25-30.
122. The vessel seen in the clinical photographs shown in Figures 50a and 50b (1,2 intercompartmental supraretinacular artery) is being dissected to be used as a source of vascularized bone graft for a patient who is scheduled to undergo internal fixation of a scaphoid nonunion. This vessel is a branch of what artery?
1- Radial
2- Ulnar
3- Median
4- Posterior interosseous
5- Anterior interosseous
PREFERRED RESPONSE: 1
DISCUSSION: The 1,2 intercompartmental supraretinacular artery is a branch of the radial artery. The vessel provides a reliable source of vascularized bone graft with an adequate pedicle length for use in scaphoid nonunions.
REFERENCES: Sheetz KK, Bishop AT, Berger RA: The arterial blood supply of the distal radius and ulna and its potential use in vascularized pedicled bone grafts. J Hand Surg 1995;20:902-914.
Steinmann SP, Bishop AT, Berger RA: Use of the 1,2 intercompartmental supraretinacular artery as a vascularized pedicle bone graft for difficult scaphoid nonunion. J Hand Surg
2002;27:391-401.
123. The flap shown in the clinical photograph seen in Figure 51 is based on what
arterial supply?
1- Superficial circumflex iliac
2- Femoral lateral accessory
3- Inferior epigastric
4- Inferior inguinal
5- Lateral obturator
PREFERRED RESPONSE: 1
DISCUSSION: The groin flap is based on the superficial circumflex iliac artery, an axial flap that has been a mainstay of providing soft-tissue coverage of the upper extremity. Flaps as large as 35 cm in length and 15 cm in width have been reported. An advantage of the flap is that when used as a pedicle flap, the donor site can be closed directly. A disadvantage of the flap is that it can be quite bulky and can have a thick layer of subcutaneous fat. The superficial circumflex iliac artery travels lateral and superficial to the fascia and below and parallel to the inguinal ligament. It is helpful to elevate the fascia at the medial border of the sartorius muscle to include the deep and superficial branches of the artery for improved flap survival.
REFERENCES: McGregor IA, Jackson IT: The groin flap. Br J Plast Surg 1972;25:3-9.
Lister GD, McGregor IA Jackson IT: The groin flap in hand injuries. Injury 1973;4:229.
124. A 63-year-old woman who sustained a distal radial fracture 2 months ago now reports that she is unable to achieve active extension of the thumb at the interphalangeal joint. What type of trauma may lead to this clinical finding?
1- Nondisplaced fracture
2- Open fracture
3- Severely comminuted fracture
4- Dorsal perilunate dislocation
5- Repeated attempts at reduction
PREFERRED RESPONSE: 1
DISCUSSION: Nondisplaced distal radial fractures have a higher rate of spontaneous rupture of the extensor pollicis longus tendon. The extensor mechanism is felt to impinge on the tendon following a nondisplaced fracture and causes either a mechanical attrition of the tendon or a local area of ischemia in the tendon.
REFERENCES: Helal B, Chen SC, Iwegbu G: Rupture of the extensor pollicis longus tendon in undisplaced Colles’ type of fracture. Hand 1982;14:41-47.
Hirasawa Y, Katsumi Y, Akiyoshi T, et al: Clinical and microangiographic studies on the rupture of the EPL tendon after distal radial fractures. J Hand Surg Br 1990;15:51-57.
125. What radiographic view will best reveal degeneration of the pisotriquetral joint in a patient who is being evaluated for pisotriquetral arthrosis?
1- True lateral
2- Lateral in 30 degrees of pronation
3- Lateral in 30 degrees of supination
4- Posteroanterior in 30 degrees of pronation
5- Carpal tunnel
PREFERRED RESPONSE: 3
DISCUSSION: The pisotriquetral joint is best seen on a lateral view in 30 degrees of supination. The carpal tunnel view provides visualization of the joint but to a lesser extent. The other views do not provide clear and accurate visualization.
REFERENCES: Paley D, McMurty RY, Cruickshank B: Pathologic conditions of the pisiform and pisotriquetral joint. J Hand Surg Am 1987;12:110-119.
Steinmann SP, Linsheid RL: Pisotriquetral loose bodies. J Hand Surg 1997;22:918-921.
related link ortho mcqs bank